Рабочая программа по математике, ФГОС 5класс



Муниципальное казённое общеобразовательное учреждение
Заброденская средняя общеобразовательная школа
Калачеевского муниципального района Воронежской области

Рассмотрена:
на заседании ШМО учителей физико- математичского цикла от 26.08.2015 г. Протокол № 1

Принята:
на заседании
педагогического совета
МКОУ Заброденская СОШ
от 27.08.2015 года
Протокол № 1
Утверждена:
приказом директора
МКОУ Заброденская СОШ № 126
от 01.09.2015г
Директор школы:
______ /Г. В. Шилова./




РАБОЧАЯ ПРОГРАММА
по МАТЕМАТИКЕ
в 5 классе
на 2015 -2016 учебный год

Программа разработана на основе примерной программы по математике федерального государственного стандарта основного общего образования.


Учитель: Данилова Татьяна Николаевна
Стаж: 24 года, категория: I КК



Согласована:
Зам директора поУВР
_______________ФИО
«____»_______________2015г



2015г.

Основные части программы

Пояснительная записка
Содержание курса математики 5 класса
Требования к уровню подготовки учащихся.
Планируемые результаты изучения учебного предмета.
Система оценки достижения планируемых результатов освоения.
Учебно-тематический план.
Календарно-тематическое планирование.
Учебно-методические средства обучения.































I. Пояснительная записка

Уровень рабочей программы – базовый
Классификация рабочей программы - типовая

Данная программа разработана на основе:

Фундаментального ядра содержания общего образования и Требований к результатам общего образования, представленных в федеральном государственном образовательном стандарте общего образования, с учетом преемственности с Примерными программами для начального общего образования. В ней также учитываются основные идеи и положения Программы развития и формирования универсальных учебных действий для основного общего образования.
Закона РФ «Об образовании».
Авторской программы по математике под редакцией Жохова В.И. составленная на основе ФГОС к УМК «Математика» для 5-6 классов, авторы Н.Я. Виленкин и др.(М.: Мнемозина). В состав УМК входят различные пособия для учащихся и учителей: контрольные работы, математические диктанты, математический тренажер, методические рекомендации для учителя, которые обеспечивают преемственность курсов математики в начальной школе и курсов алгебры в последующих классах для большинства программ, позволяют проводить разноуровневое обучение и качественную подготовку школьников к изучению курсов алгебры и геометрии (в том числе стереометрии) в старших классах, а также смежных дисциплин физики, химии, географии и др. Учебник содержит разнообразные упражнения к каждому параграфу. Среди них: задания, связанные с закреплением изученного материала, задачи повышенной трудности, занимательные и развивающие упражнения, некоторые упражнения из учебника с пояснениями, иллюстрациями, образцами выполнения заданий, помогающими учащимся лучше понять их содержание. Состоит в федеральном перечне. Рекомендован Министерством образования и науки Российской Федерации.
Учебного плана МКОУ Заброденская СОШ.



Нормативно-правовые документы, на основании которых разработана рабочая программа
Приказ Министерства образования и науки Российской Федерации № 1997 от 17 декабря 2010 г. «Федеральный государственный образовательный стандарт основного общего образования».
Приказ Министерства образования Российской Федерации № 1312 от 09.03.2004 «Об утверждении федерального базисного учебного плана и примерных планов для образовательных учреждений Российской Федерации, реализующих программы общего образования».
Приказа Министерства образования и науки Российской Федерации № 889 от 30.08.2010 «О внесении изменений в федеральный базисный учебный план и примерные учебные планы для образовательных учреждений Российской Федерации, реализующих программы общего образования, утвержденные приказом Министерства образования Российской Федерации № 1312 от 09.03.2004 «Об утверждении федерального базисного учебного плана и примерных планов для образовательных учреждений Российской Федерации, реализующих программы общего образования».
Постановления Главного государственного санитарного врача Российской Федерации № 189 от 29.12.2010 «Об утверждении СанПин 2.4.2.2821-10 «Санитарно-эпидемиологические требования к условиям и организации обучения в общеобразовательных учреждениях» (зарегистрировано в Минюсте России 03.03.2011, регистрационный номер .19993).
Приказ Министерства образования и науки Российской Федерации № 373 от 06.10.2009 г «Об утверждении и введении в действие федерального государственного образовательного стандарта начального общего образования» (зарегистрирован Минюстом России 22.12.2009, регистрационный №15785).
Приказ Министерства образования и науки Российской Федерации от 26.11.2010 г № 1241 «О внесении изменений в федеральный стандарт начального общего образования, утвержденный приказом Минобрнауки России от 06.10.2009 №373» (зарегистрирован Минюстом России. 03.03.2011, регистрационный номер 19993).
Приказ «Об утверждении регионального базисного учебного плана для образовательных учреждений Воронежской области, реализующих программы общего образования в новой редакции» № 441 от 20.05.2011.
«Федеральный перечень учебников, рекомендованных (допущенных) к использованию в образовательном процессе в общеобразовательных учреждениях, реализующих образовательные программы общего образования и имеющих аккредитацию на 2015/2016 учебный год», утвержденный приказом Министерства образования Российской Федерации № 2885 от 17.12.2011г.

[ Cкачайте файл, чтобы посмотреть ссылку ]













Психолого-педагогическая характеристика 5 класса

В классе 9 человек. Из них 5 мальчик и 4 девочек.
Коллектив составлен из учащихся, которые в начальной школе учились в одном классе.
Уровень математических способностей, навыки учебной деятельности, познавательный интерес у учащихся класса различны, поэтому работать с классом будет не просто. Необходимо постоянно этот факт учитывать при подготовке учителя к уроку.
Пивторак Вика, Андреева Евгения, Колотев Максим, Сливкина Даша – по диагностике в числе лучших по математике. Умны, активны, ответственны, им нравится сам процесс познания нового.
Три ученика, у которых результат в учёбе наблюдается средний. Это Маметьев Денис, Шушпанов Сергей, Камынин Андрей. Им непросто усваивать новый материал и, соответственно, применять его при решении даже несложных заданий.
Необходимо постоянно на уроке держать в поле зрения Варю. «Непоседа», для неё характерен дефицит внимания. Заранее необходимо продумывать приёмы поддержания внимания в рабочем состоянии на протяжении урока.
Интересы: ученики имеют схожие интересы. Большинство слушает одну и ту же музыку, увлекается похожими фильмами и телепередачами. Особенно активно дети обсуждают новые модели мобильных телефонов.
В свободное от учебы время каждый ученик находит себе занятие, посещают кружки при школе.
В целом, в классе благоприятная обстановка, доброжелательная и дружелюбная атмосфера. Конфликтов между одноклассниками нет. К учителям дети относятся с уважением.

Рабочая программа составлена с учетом возрастных и индивидуальных особенностей обучающихся 5класса.



Общая характеристика рабочей программы по математике для 5 класса.

Рабочая программа по математике 5 класса «Математика» содержит перечень вопросов, которые подлежат обязательному изучению в 5 классе. В ней сохранена традиционная ориентация на фундаментальный характер образования, на освоение учениками основополагающих понятий и идей, и включает материал, создающий основы математической грамотности.
При этом программа является достаточно динамичной:
увеличены активные формы работы, направленные на вовлечение учащихся в математическую деятельность,
на обеспечение понимания ими математического материала и развития интеллекта, приобретения практических навыков, умений проводить рассуждения, доказательства.
наряду с этим в ней уделяется внимание использованию компьютеров и информационных технологий.
Содержание раздела «Арифметика» служит базой для дальнейшего изучения учащимися математики, способствует развитию их логического мышления, формированию умения пользоваться алгоритмами, а также приобретению практических навыков, необходимых в повседневной жизни.
Программа реализуется с помощью современных педагогических технологий: технологии уровневой дифференциации, разноуровнего обучения, групповых технологий, а также современного традиционного обучения, использующее классическую систему в сочетании с техническими средствами.
Каждая из этих технологий предусматривает организацию учебного процесса через теоретические и практические виды занятий.
Задачами изучения курса математики в 5 классе является систематическое развитие понятия числа, выработка умений выполнять устно и письменно арифметические действия над числами, переводить практические задачи на язык математики, подготовка учащихся к изучению систематических курсов алгебры и геометрии.
Курс, строится на индуктивной основе с привлечением элементов дедуктивных рассуждений. Теоретический материал курса излагается на наглядно-интуитивном уровне, математические методы и законы формулируются в виде правил.
В ходе изучения курса учащиеся развивают навыки вычислений с натуральными числами. Овладевают навыками действий с обыкновенными и десятичными дробями, положительными и отрицательными числами, получают начальные представления об использовании букв для записи выражений и свойств арифметических действий, составление уравнений, продолжают знакомство с геометрическими понятиями, приобретают навыки построения геометрических фигур и измерения геометрических величин.





Цели изучения математики в 5 классе

в направлении личностного развития
развитие логического и критического мышления, культуры речи, способности к умственному эксперименту;
формирование у учащихся интеллектуальной честности и объективности, способности к преодолению мыслительных стереотипов, вытекающих из обыденного опыта;
воспитание качеств личности, обеспечивающих социальную мобильность, способность принимать самостоятельные решения;
формирование качеств мышления, необходимых для адаптации в современном информационном обществе;
развитие интереса к математическому творчеству и математических способностей;
в метапредметном направлении
формирование представлений о математике как части общечеловеческой культуры, о значимости математики в развитии цивилизации и современного общества;
развитие представлений о математике как форме описания и методе познания действительности, создание условий для приобретения первоначального опыта математического моделирования;
формирование общих способов интеллектуальной деятельности, характерных для математики и являющихся основой познавательной культуры, значимой для различных сфер человеческой деятельности;
в предметном направлении
овладение математическими знаниями и умениями, необходимыми для продолжения обучения в старшей школе или иных общеобразовательных учреждениях, изучения смежных дисциплин, применения в повседневной жизни;
создание фундамента для математического развития, формирования механизмов мышления, характерных для математической деятельности.

Рабочая программа рассчитана:
Количество недельных часов – 5
Количество часов в год – 175
Количество контрольных работ (за год) – 15











II. Содержание программы 5 класса.

(5 ч в неделю, всего 175 ч.)

Повторение изученного в начальной школе. 3 ч
Натуральные числа и шкалы. 16 ч.
Сложение и вычитание натуральных чисел. 20 ч.
Умножение и деление натуральных чисел. 22 ч.
Площади и объемы. 14 ч.
Обыкновенные дроби. 20 ч.
Десятичные дроби. Сложение и вычитание десятичных дробей. 15 ч.
Умножение и деление десятичных дробей. 20 ч.
Инструменты для вычислений и измерений. 20 ч.
Описательная статистика. Вероятность. Комбинаторика. 5 ч.
Повторение. Решение задач. 20ч.

1.Натуральные числа и шкалы.
Натуральные числа и их сравнение. Геометрические фигуры: отрезок, прямая, луч, треугольник. Измерение и построение отрезков. Координатный луч.
Основная цель - систематизировать и обобщить сведения о натуральных числах, полученных в начальной школе; закрепить навыки построения и измерения отрезков.
Систематизация сведений о натуральных числах позволяет восстановить у учащихся навыки чтения и записи многозначных чисел, сравнения натуральных чисел, а также навыки измерения и построения отрезков.
В ходе изучения темы вводятся понятия координатного луча, единичного отрезка и координаты точки. Здесь начинается формирование таких важных умений, как умения начертить координатный луч и отметить на нем заданные числа, назвать число, соответствующее данному делению на координатном луче.

2. Сложение и вычитание натуральных чисел.
Сложение и вычитание натуральных чисел, свойство сложения. Решение текстовых задач. Числовое выражение. Буквенное выражение и его числовое значение. Решение линейных уравнений.
Основная цель – закрепить и развить навыки сложения и вычитания натуральных чисел.
Начиная с этой темы, основное внимание уделяется закреплению алгоритмов арифметических действий над многозначными числами, так как они не только имеют самостоятельное значение, но и являются базой для формирования умений проводить вычисления с десятичными дробями.
В этой теме начинается алгебраическая подготовка: составление буквенных выражений по условию задач, решение уравнений на основе зависимости между компонентами действий (сложение и вычитание).
3. Умножение и деление натуральных чисел.
Умножение и деление натуральных чисел, свойства умножения. Квадрат и куб числа. Решение текстовых задач.
Основная цель – закрепить и развить навыки арифметических действий с натуральными числами.
В этой теме проводится целенаправленное развитие и закрепление навыков умножения и деления многозначных чисел. Вводятся понятия квадрата и куба числа. Продолжается работа по формированию навыков решения уравнений на основе зависимости между компонентами действий. Развиваются умения решать текстовые задачи, требующие понимания смысла отношений «больше на (в)», «меньше на (в)», а также, умения решать задачи на известные учащимся зависимости между величинами (скоростью, временем и расстоянием; ценой, количеством и стоимостью товара и др.). Задачи решаются арифметическим способом. При решении с помощью составления уравнений задач на части, учащиеся впервые встречаются с уравнениями, в левую часть которых неизвестное входит дважды. Решению таких задач предшествуют преобразования соответствующих буквенных выражений.
4.Обыкновенные дроби.
Окружность и круг. Обыкновенная дробь. Основные задачи на дроби. Сравнение обыкновенных дробей. Сложение и вычитание дробей с одинаковыми знаменателями.
Основная цель – познакомить учащихся с понятием дроби в объеме, достаточном для введения десятичных дробей.
В данной теме изучаются сведения о дробных числах, необходимые для введения десятичных дробей. Среди формируемых умений основное внимание должно быть привлечено к сравнению дробей с одинаковыми знаменателями, к выделению целой части числа. С пониманием смысла дроби, осознанного решения которых важно добиться от учащихся.
5. Десятичные дроби. Сложение и вычитание десятичных дробей.
Десятичная дробь. Сравнение, округление, сложение и вычитание десятичных дробей. Решение текстовых задач.
Основная цель – выработать умения читать, записывать, сравнивать, округлять десятичные дроби, выполнять сложение и вычитание десятичных дробей.
При введении десятичных дробей важно добиться у учащихся четкого представления о десятичных разрядах рассматриваемых чисел, умений читать, записывать, сравнивать десятичные дроби.
Подчеркивая сходство действий над десятичными дробями с действиями над натуральными числами, отмечается, что сложение десятичных дробей подчиняется переместительному и сочетательному законам.
Определенное внимание уделяется решению текстовых задач на сложение и вычитание, данные в которых выражены десятичными дробями.
При изучении операции округления числа вводится новое понятие – «приближенное значение числа», отрабатываются навыки округления десятичных дробей до заданного десятичного разряда.


6. Умножение и деление десятичных дробей.
Умножение и деление десятичных дробей. Среднее арифметическое нескольких чисел.
Решение текстовых задач.
Основная цель – выработать умения умножать и делить десятичные дроби, выполнять задания на все действия с натуральными числами и десятичными дробями.
Основное внимание привлекается к алгоритмической стороне рассматриваемых вопросов. На несложных примерах отрабатывается правило подстановки запятой в результате действия. Кроме того, продолжается решение текстовых задач с данными, выраженными десятичными дробями. Вводится понятие среднего арифметического нескольких чисел.
7. Площади и объемы.
Вычисления по формулам. Прямоугольник. Площадь прямоугольника. Единицы площадей.
Основная цель – расширить представления учащихся об измерении геометрических величин на примере вычисления площадей и объёмов и систематизировать известные им сведения о единицах измерения.
При изучении темы учащиеся встречаются с формулами. Навыки вычисления по формулам отрабатываются при решении геометрических задач. Значительное внимание уделяется формированию знаний основных единиц измерения и умению перейти от одних единиц к другим в соответствии с условием задачи
8. Инструменты для вычислений и измерений.
Начальные сведения о вычислениях на калькуляторе. Проценты. Основные задачи на проценты. Примеры таблиц и диаграмм. Угол, треугольник. Величина (градусная мера) угла. Единицы измерения углов. Измерение углов. Построение угла заданной величины.
Основная цель – сформировать умения решать простейшие задачи на проценты, выполнять измерение и построение углов.
У учащихся важно выработать содержательное понимание смысла термина «процент». На этой основе они должны научиться решать три вида задач на проценты: находить несколько процентов от какой-либо величины; находить число, если известно несколько его процентов; находить, сколько процентов одно число составляет от другого.
Продолжается работа по распознаванию и изображению геометрических фигур. Важно уделить внимание формированию умений проводить измерения и строить углы.
Круговые диаграммы дают представления учащимся о наглядном изображении распределения отдельных составных частей какой-нибудь величины. В упражнениях следует широко использовать статистический материал, публикуемый в газетах и журналах.
В классе, обеспеченном калькуляторами, можно научить школьников использовать калькулятор при выполнении отдельных арифметических действий.


9. Описательная статистика. Вероятность. Комбинаторика.
Достоверные, невозможные и случайные события. Перебор вариантов, дерево вариантов. Комбинаторные задачи.
Основная цель – научить проводить несложные доказательства, получать простейшие следствия из известных или ранее полученных утверждений, оценивать логическую правильность рассуждений, использовать примеры для иллюстрации и контрпримеры для опровержения утверждений;
извлекать информацию, представленную в таблицах, на диаграммах, графиках; составлять таблицы, строить диаграммы и графики;
решать комбинаторные задачи путем систематического перебора возможных вариантов, а также с использованием правила умножения;
вычислять средние значения результатов измерений;
находить частоту события, используя собственные наблюдения и готовые статистические данные; находить вероятности случайных событий в простейших случаях;


10.Повторение. Решение задач.
Основная цель – повторить и систематизировать знания учащихся полученные в результате изучения курса математики 5 класса.


























III. Требования к уровню подготовки учащихся

в направлении личностного развития:
уметь ясно, точно, грамотно излагать свои мысли в устной и письменной речи, понимать смысл поставленной задачи, выстраивать аргументацию, приводить примеры и контрпримеры; (л1)
, уметь распознавать логически некорректные высказывания, отличать гипотезу от факта; критичность мышления (л2)
иметь представление о математической науке как сфере человеческой деятельности, об этапах ее развития, о ее значимости для развития цивилизации; (л3)
креативность мышления, инициатива, находчивость, активность при решении математических задач; (л4)
уметь контролировать процесс и результат учебной математической деятельности; (л5)
способность к эмоциональному восприятию математических объектов, задач, решений, рассуждений. (л6)

в метапредметном направлении
первоначальные представления об идеях и о методах математики как об универсальном языке науки и техники, о средстве моделирования явлений и процессов; (м1)
уметь видеть математическую задачу в контексте проблемной ситуации в других дисциплинах, в окружающей жизни; (м2)

уметь находить в различных источниках информацию, необходимую для решения математических проблем, и представлять ее в понятной форме; принимать решение в условиях неполной и избыточной, точной и вероятностной информации; (м3)

уметь понимать и использовать математические средства наглядности (графики, диаграммы, таблицы, схемы и др.) для иллюстрации, интерпретации, аргументации; (м4)

уметь выдвигать гипотезы при решении учебных задач и понимать необходимость их проверки; (м5)

уметь применять индуктивные и дедуктивные способы рассуждений, видеть различные стратегии решения задач; (м6)

понимать сущность алгоритмических предписаний и умение действовать в соответствии с предложенным алгоритмом; (м7)

уметь самостоятельно ставить цели, выбирать и создавать алгоритмы для решения учебных математических проблем; (м8)

уметь планировать и осуществлять деятельность, направленную на решение задач исследовательского характера. (м9)


В предметном направлении
овладеть базовым понятийным аппаратом по основным разделам содержания; представление об основных изучаемых понятиях (число, геометрическая фигура, уравнение, вероятность) как важнейших математических моделях, позволяющих описывать и изучать реальные процессы и явления; (п1)

уметь работать с математическим текстом (анализировать, извлекать необходимую информацию), точно и грамотно выражать свои мысли в устной и письменной речи с применением математической терминологии и символики, использовать различные языки математики, проводить классификации, логические обоснования, доказательства математических утверждений; (п2)

развивать представлений о числе и числовых системах от натуральных до действительных чисел; овладеть навыками устных, письменных, инструментальных вычислений; (п3)

овладеть основными способами представления и анализа статистических данных; наличие представлений о статистических закономерностях в реальном мире и о различных способах их изучения, о вероятностных моделях; (п4)

овладеть геометрическим языком, умение использовать его для описания предметов окружающего мира; развитие пространственных представлений и изобразительных умений, приобретение навыков геометрических построений; (п5)

уметь измерять длины отрезков, величины углов, использовать формулы для нахождения периметров, площадей и объемов геометрических фигур; (п6)

уметь применять изученные понятия, результаты, методы для решения задач практического характера и задач из смежных дисциплин с использованием при необходимости справочных материалов, калькулятора, компьютера. (п7)










IV. Планируемые результаты изучения учебного предмета

В результате изучения курса математики в 5 классе учащиеся должны
знать/понимать:
как используются математические формулы и уравнения при решении математических и практических задач;
как потребности практики привели математическую науку к необходимости расширения понятия числа;
каким образом геометрия возникла из практических задач землемерия;

уметь:
выполнять устно действия сложения и вычитания двузначных чисел и десятичных дробей с двумя знаками, умножение однозначных чисел, сложение и вычитание обыкновенных дробей с однозначным числителем и знаменателем;
переходить от одной формы записи чисел к другой, представлять десятичную дробь в виде обыкновенной и в простейших случаях обыкновенную в виде десятичной, проценты в виде дроби и дробь в виде процентов;
находить значение числовых выражений;
округлять натуральные числа и десятичные дроби, находить приближенные значения с недостатком и с избытком;
пользоваться основными единицами длины, массы, времени, скорости, площади, объема;
выражать более крупные единицы через более мелкие и наоборот;
решать текстовые задачи арифметическим способом, включая задачи, связанные с дробями и процентами.
составлять таблицы, строить диаграммы и графики;

решать комбинаторные задачи путем систематического перебора возможных вариантов.


Использовать приобретенные знания и умения в практической деятельности и повседневной жизни:
для решения несложных практических задач, в том числе с использованием справочных материалов, калькулятора, компьютера;
устной прикидки и оценки результатов вычислений; проверки результатов вычислений с использованием различных приемов V. Система оценки достижений планируемых результатов освоения. 
1. Оценка письменных контрольных работ обучающихся по математике. 
Ответ оценивается отметкой «5», если:
работа выполнена полностью;
в логических рассуждениях и обосновании решения нет пробелов и ошибок;
в решении нет математических ошибок (возможна одна неточность, описка, которая не является следствием незнания или непонимания учебного материала).
Отметка «4» ставится в следующих случаях:
работа выполнена полностью, но обоснования шагов решения недостаточны (если умение обосновывать рассуждения не являлось специальным объектом проверки);
допущены одна ошибка или есть два – три недочёта в выкладках, рисунках, чертежах или графиках (если эти виды работ не являлись специальным объектом проверки).
Отметка «3» ставится, если:
допущено более одной ошибки или более двух – трех недочетов в выкладках, чертежах или графиках, но обучающийся обладает обязательными умениями по проверяемой теме.

Учитель может повысить отметку за оригинальный ответ на вопрос или оригинальное решение задачи, которые свидетельствуют о высоком математическом развитии обучающегося; за решение более сложной задачи или ответ на более сложный вопрос, предложенные обучающемуся дополнительно после выполнения им каких-либо других заданий.

 2. Оценка устных ответов обучающихся по математике
 
Ответ оценивается отметкой «5», если ученик:
полно раскрыл содержание материала в объеме, предусмотренном программой и учебником;
изложил материал грамотным языком, точно используя математическую терминологию и символику, в определенной логической последовательности;
правильно выполнил рисунки, чертежи, графики, сопутствующие ответу;
показал умение иллюстрировать теорию конкретными примерами, применять ее в новой ситуации при выполнении практического задания;
продемонстрировал знание теории ранее изученных сопутствующих тем, сформированность и устойчивость используемых при ответе умений и навыков;
отвечал самостоятельно, без наводящих вопросов учителя;
возможны одна – две неточности при освещение второстепенных вопросов или в выкладках, которые ученик легко исправил после замечания учителя.

Ответ оценивается отметкой «4», если удовлетворяет в основном требованиям на оценку «5», но при этом имеет один из недостатков:
в изложении допущены небольшие пробелы, не исказившее математическое содержание ответа;
допущены один – два недочета при освещении основного содержания ответа, исправленные после замечания учителя;
допущены ошибка или более двух недочетов при освещении второстепенных вопросов или в выкладках, легко исправленные после замечания учителя.
 
Отметка «3» ставится в следующих случаях:
неполно раскрыто содержание материала (содержание изложено фрагментарно, не всегда последовательно), но показано общее понимание вопроса и продемонстрированы
умения, достаточные для усвоения программного материала (определены «Требованиями к математической подготовке учащихся» в настоящей программе по математике);
имелись затруднения или допущены ошибки в определение математической терминологии, чертежах, выкладках, исправленные после нескольких наводящих вопросов учителя;
ученик не справился с применением теории в новой ситуации при выполнении практи ческого задания, но выполнил задания обязательного уровня сложности по данной теме;
при достаточном знании теоретического материала выявлена недостаточная
сформированность основных умений и навыков.
3. Общая классификация ошибок.
При оценке знаний, умений и навыков учащихся следует учитывать все ошибки (грубые и негрубые) и недочёты.
3.1. Грубыми считаются ошибки:
незнание определения основных понятий, законов, правил, основных положений теории, незнание формул, общепринятых символов обозначений величин, единиц их измерения;
незнание наименований единиц измерения;
неумение выделить в ответе главное;
неумение применять знания, алгоритмы для решения задач;
неумение делать выводы и обобщения;
неумение читать и строить графики;
неумение пользоваться первоисточниками, учебником и справочниками;
потеря корня или сохранение постороннего корня;
отбрасывание без объяснений одного из них;
равнозначные им ошибки;
вычислительные ошибки, если они не являются опиской;
логические ошибки.
3.2. К негрубым ошибкам следует отнести:
неточность формулировок, определений, понятий, теорий, вызванная неполнотой охвата основных признаков определяемого понятия или заменой одного - двух из этих признаков второстепенными;
неточность графика;
нерациональный метод решения задачи или недостаточно продуманный план ответа (нарушение логики, подмена отдельных основных вопросов второстепенными);
нерациональные методы работы со справочной и другой литературой;
неумение решать задачи, выполнять задания в общем виде.

3.3. Недочетами являются:
нерациональные приемы вычислений и преобразований;
небрежное выполнение записей, чертежей, схем, графиков. 
При оценке выполнения тестового задания используется следующая шкала
 
Баллы
Степень выполнения заданий

1
Менее чем на балл «2»

2
Выполнено не менее 20 % предложенных заданий

3
Выполнено не менее 30 % предложенных заданий      

4
Выполнено не менее 40 % предложенных заданий

5
Выполнено не менее 50 % предложенных заданий

6
Выполнено не менее 60 % предложенных заданий

7
Выполнено не менее 70 % предложенных заданий

8
Выполнено не менее 80 % предложенных заданий

9
Выполнено не менее 90 % предложенных заданий  

10
Выполнены все предложенные задания

VI. Учебно-тематический план

Количество недельных часов – 5
Количество часов в год – 175
Программа – автор: Н.Я. Виленкин
Учебник: Математика. 5 класс: учеб. для общеобразоват. учреждений/ Н.Я. Виленкин и др. – 24-е изд., стер. – М.: Мнемозина, 2010г. – 208 с.
Автор: Н.Я. Виленкин
Количество контрольных работ (за год) – 15


Разделы курса
Количество
часов
Теория
(часов)
Практика
(часов)
Контрольные
работы

Повторение курса начальной школы
3
1
2



Натуральные числа и шкалы.
16
2
13
1

Сложение и вычитание натуральных чисел
20
2
16
2

Умножение и деление натуральных чисел
22
3
16
3

Площади и объёмы
14
2
10
2

Обыкновенные дроби
20
2
17
1

Десятичные дроби. Сложение и вычитание десятичных дробей.
15
3
10
2

Умножение и деление десятичных дробей.
20
3
15

2

Инструменты для вычислений и измерений.
20
3
16
1

Описательная статистика. Вероятность. Комбинаторика
5
1
4
0

Итоговое повторение курса математики 5 класса
20
2
17
1







Всего
175
24
136
15














№ п
Основное содержание по темам
Кол-во часов

Характеристика основных видов деятельности ученика (на уровне учебных действий)


Повторение изученного начальной школе.
3



§1. Натуральные числа и шкалы.
16
Описывать свойства натурального ряда.
Читать и записывать натуральные числа, сравнивать и упорядочивать их.
Выполнять вычисления с натуральными числами; вычислять значения степеней.
Формулировать свойства арифметических действий, записывать их с помощью букв, преобразовывать на их основе числовые выражения.
Анализировать и осмысливать текст задачи, переформулировать условие, извлекать необходимую информацию, моделировать условие с помощью схем, рисунков, реальных предметов; строить логическую цепочку рассуждений; критически оценивать полученный ответ на соответствие условию.
Исследовать простейшие числовые закономерности, проводить числовые эксперименты (в том числе с использованием калькулятора, компьютера).
Читать и записывать буквенные выражения, составлять буквенные выражения по условию задач. Вычислять числовое значение буквенного выражения при заданных значениях букв.
Составлять уравнения по условиям задач. Решать простейшие уравнения на основе зависимостей между компонентами арифметических действий.

1
Натуральных числа
1


2
Отрезок. Длина отрезка. Треугольник
2


3
Плоскость, прямая, луч
2


4
Шкалы и координаты
2


5
Меньше или больше
2



6
Натуральные числа и шкалы
3


7
Контрольная работа
1



§2. Сложение и вычитание натуральных чисел
20 ч


6
Сложение натуральных чисел и его свойства
5


7
Вычитание
4


8
Числовые и буквенные выражения
3


9
Буквенная запись свойств сложения и вычитания
3


10
Уравнение
6



§ 3. Умножение и деление натуральных чисел
22 ч


11
Умножение натуральных чисел и его свойства
5


12
Деление
5


13
Деление с остатком
2


14
Упрощение выражений
7


15
Порядок выполнения действий
3


16
Квадрат и куб
5



§ 4. Площади и объёмы
14
Выражать одни единицы измерения величины в других единицах (метры в километрах, минуты в часах и т.д.).
Моделировать несложные зависимости с помощью формул, выполнять вычисления по формулам.
Использовать знания о зависимостях между величинами (производительность, время, работа, цена, количество, стоимость, скорость, время, расстояние), при решении текстовых задач.
Распознавать на чертежах, рисунках, в окружающем мире геометрические фигуры, конфигурации фигур(плоские и пространственные).Приводить примеры аналогов геометрических фигур в окружающем мире.
Изображать геометрические фигуры с использованием чертежных инструментов.
Изображать геометрические фигуры на клетчатой бумаге.
Вычислять площади квадратов и прямоугольников, используя формулы площади квадрата и прямоугольника. Выражать одни единицы измерения площади через другие.
Изготавливать пространственные фигуры из разверток; распознавать развертки куба, параллелепипеда, пирамиды, цилиндра, конуса.
Вычислять объёмы куба, прямоугольного параллелепипеда. Выражать одни единицы измерения объёма через другие.

17
Формулы
2


18
Площадь. Формула площади прямоугольника
3


19
Единицы измерения площадей
3


20
Прямоугольный параллелепипед
3


21
Объёмы прямоугольного параллелепипеда
3



§ 5. Обыкновенные дроби
20
Моделировать в графической, предметной форме понятия и свойства, связанные с понятием обыкновенной дроби.
Формулировать, записывать с помощью букв основное свойство дроби, правила действий с обыкновенными дробями.
Преобразовывать обыкновенные дроби, сравнивать и упорядочивать их. Выполнять вычисления с обыкновенными дробями.

22
Окружность и круг
2


23
Доли. Обыкновенные дроби
3


24
Сравнение дробей
2


25
Правильные и неправильные дроби
2


26
Сложение и вычитание дробей с одинаковыми знаменателями
3


27
Деление и дроби
2


28
Смешанные числа
3


29
Сложение и вычитание смешанных чисел
4



§ 6. Десятичные дроби. Сложение и вычитание десятичных дробей.
15
Записывать и читать десятичные дроби. Представлять десятичные дроби в виде обыкновенных дробей и обыкновенные в виде десятичных. Находить десятичные приближения обыкновенных дробей. Сравнивать и упорядочивать десятичные дроби. Выполнять вычисления с десятичными дробями.
Выполнять прикидку и оценку вычислений.
Объяснять, что такое процент. Представлять проценты в дробях и дроби в виде процентах.
Решать задачи на проценты и дроби (в том числе задачи из реальной практики, используя при необходимости калькулятор).
Анализировать и осмысливать текст задачи, переформулировать условие, извлекать необходимую информацию, моделировать условие с помощью схем, рисунков, реальных предметов; строить логическую цепочку рассуждений; критически оценивать полученный ответ на соответствие условию.
Проводить несложные исследования, связанные со свойствами дробных чисел, опираясь на числовые эксперименты (в том числе с использованием калькулятора и компьютера).

30
Десятичная запись дробных чисел
3


31
Сравнение десятичных дробей
2


32
Сложение и вычитание десятичных дробей
6


33
Приближённые значения чисел. Округление чисел
4



§ 7. Умножение и деление десятичных дробей.
20


34
Умножение десятичных дробей на натуральные числа
3


35
Деление десятичных дробей на натуральные числа
5


36
Умножение десятичных дробей
5


37
Деление десятичных дробей
5


38
Среднее арифметическое
2



§ 8. Инструменты для вычислений и измерений.
20
Измерять с помощью инструментов и сравнивать длины отрезков и величины углов. Строить отрезки заданной длины с помощью линейки и циркуля и углы заданной величины с помощью транспортира. Выражать одни единицы измерения длин через другие.

39
Микрокалькулятор
2


40
Проценты
5


41
Угол. Прямой и развёрнутый угол. Чертёжный треугольник
4


42
Измерение углов. Транспортир
5


43
Круговые диаграммы
4



Описательная статистика. Вероятность. Комбинаторика
5ч.

Формировать представление о достоверных, невозможных, случайных событиях.
Овладеть умением составлять дерево возможных вариантов
Овладеть навыками решения простейших комбинаторных задач.

44
Введение в вероятность. Перебор возможных вариантов
1


45
Дерево возможных вариантов
2


46
Достоверные, невозможные и случайные события
1


47
Решение простейших комбинаторных задач
1



Итоговое повторение курса математики 5 класса
20









Перечень контрольных работ


Тема контрольной работы
Дата

Контрольная работа № 1
Административная контрольная работа
21.09-26.09

Контрольная работа № 2
Натуральные числа и шкалы
05.10-10.10

Контрольная работа № 3
Сложение и вычитание натуральных чисел
19.10-24.10

Контрольная работа № 4
Числовые и буквенные выражения. Уравнения
16.11-21.11

Контрольная работа № 5
Умножение и деление натуральных чисел
07.12-12.12

Контрольная работа № 6
Арифметические действия с натуральными числами
21.12-26.12

Контрольная работа № 7
Административная контрольная работа.
25.01-30.01

Контрольная работа № 8
Обыкновенные дроби
08.02-13.02

Контрольная работа № 9
Сложение и вычитание обыкновенных дробей. Смешанные числа.
14.03-19.03

Контрольная работа № 10
Десятичные дроби. Сложение и вычитание десятичных дробей
29.03-02.04

Контрольная работа № 11
Умножение и деление десятичных дробей на натуральное число.
11.04-16.04

Контрольная работа № 12
Умножение и деление десятичных дробей.
25.04-30.04

Контрольная работа № 13
Проценты
03.05-10.05

Контрольная работа № 14
Инструменты для вычислений и измерений
18.05-23.05

Контрольная работа № 15
Итоговая административная контрольная работа
27.05-30.05







VII. Календарно - тематическое планирование.
№ п.п
Тема
урока

Д/З
УУД

1
Повторение курса математики начальной школы
Числа и величины. Арифметические действия
п1, № 17
л.2, м1,м3, п1,п7

2
Повторение курса математики начальной школы.
Геометрические фигуры. Геометрические величины.

п.1, № 19,20
л.2, м1,м3, п1,п7

3
Повторение курса математики начальной школы.
Текстовые задачи.

тест
л.2, м1,м3, п1,п7


§1. НАТУРАЛЬНЫЕ ЧИСЛА И ШКАЛЫ (16 часов)


4
Натуральные числа.


п.1 правило, № 23,25, 30
л.2, м1,м2, п1,п7

5
Отрезок. Длина отрезка. Треугольник.

п.2, № 65,68
л.2, м9,м3, п1,п7

6
Отрезок. Длина отрезка. Треугольник.

п.2, № 66,72
л.2, м1,м2, п1,п7

7
Плоскость. Прямая. Луч.

п.3, № 99,101, 103
П.7, м3, л4

8
Плоскость. Прямая. Луч.

п.3 № 100,106
л.2, м1,м2, п1,п7

9
Шкалы и координаты.

п.4, № 137, 141
П7, л1,л5, м7

10
Шкалы и координаты.


п.4, № 138, 139, 144 (а,в)
л.2, м1,м2, п1,п7

11
Меньше или больше.


п.5, № 168, 169
П7, л1,л5, м7

12
Меньше или больше.
п.5, №172, 180
л.2, м1,м3, п1,п7

13
Натуральные числа и шкалы

карточка
П7, л1,л5, м7

14

Натуральные числа и шкалы
тест
л.2, м1,м2, п1,п7

15
Натуральные числа и шкалы

карточка
П.7, м3, л4

16
Контрольная работа № 1 по теме «Натуральные числа и шкалы»


п.2,м.8.л1,л5

17
Анализ контрольной работы. Как возникла арифметика
Сообщение по теме
л.2, м1,м3, п1,п7

18
Входное тестирование.


л.2, м1,м2, п1,п7

19
Рефлексия
тест
П.7, м3, л4


§2. СЛОЖЕНИЕ И ВЫЧИТАНИЕ НАТУРАЛЬНЫХ ЧИСЕЛ (20 час)


20
Сложение натуральных чисел и его свойства.
п.6, № 231, 232
П7, л1,л5, м7

21
Сложение натуральных чисел и его свойства.
п.6, № 237, 240
П.7, м3, л4

22
Вычитание.
п.7, № 288, 291
П7, л1,л5, м7

23
Вычитание.

п. 7 № 290, 292
П.7 п.2,м.8.л1,л5, м3, л4

24
Свойства сложения и вычитания натуральных чисел
п. 6,7 № 296, 295
п.2,м.8.л1,л5

25
Свойства сложения и вычитания натуральных чисел
п.6,7 № 231, 294
П7, л1,л5, м7

26
Контрольная работа по теме №2 Сложение и вычитание натуральных чисел

п.2,м.8.л1,л5

27
Числовые и буквенные выражения.
п. 8 №328, 331
П.7, м3, л4

28
Числовые и буквенные выражения.
п. 8 №335
П7, л1,л5, м7

29
Буквенная запись свойств сложения и вычитания.
п. 9 №364, 371
П.7, м3, л4

30
Буквенная запись свойств сложения и вычитания.
п. 9 №365, 368
п.2,м.8.л1,л5

31
Уравнения.

п.10 №395, 397
П7, л1,л5, м7

32
Уравнения.

п. 10 № 396, 308
П.8, м3, л4

33
Уравнения.
п. 10 №399, 403
П7, л1,л5, м7

34
Сложение и вычитание натуральных чисел
карточка
П.7, м3, л4

35
Сложение и вычитание натуральных чисел
карточка
П7, л1,л5, м7

36
Сложение и вычитание натуральных чисел
тест
П7, л1,л5, м7

37
Сложение и вычитание натуральных чисел
тест
П.7, м3, л4

38
Контрольная работа № 3 по теме «Числовые и буквенные выражения. Уравнения»

п.2,м.8.л1,л5

39
Анализ контрольной работы. Как возникла арифметика. Натуральные числа в природе
Подготовить сообщение.
П7, л1,л5, м7


§3. УМНОЖЕНИЕ И ДЕЛЕНИЕ НАТУРАЛЬНЫХ ЧИСЕЛ.(22 часов)

40
Умножение натуральных чисел и его свойств.
п. 11 № 450, 452
П7, л1,л5, м7

41
Умножение натуральных чисел и его свойств.
п. 11 №460, 461
п.2,м.8.л1,л5

42
Деление.
п. 12 №516, 517
П7, л1,л5, м7

43
Деление.
п. 12 №522,524


44
Деление с остатком.
п. 13 №
П7, л1,л5, м7

45
Деление с остатком.
п. №
п.2,м.8.л1,л5

46
Умножение и деление натуральных чисел
п. №
П.3, п.2, п.7, л.1, м.7

47
Умножение и деление натуральных чисел
п. №
П7, л1,л5, м7

48
Умножение и деление натуральных чисел
п. №
П.3, п.2, п.7, л.1, м.7

49
Контрольная работа № 4 «Умножение натуральных чисел»

П7, л1,л5, м7

50
Упрощение выражений
п. №
Л.1,л2, п2, м8

51
Упрощение выражений.
п. №
П7, л1,л5, м7

52
Упрощение выражений.
п. №
П.3, п.2, п.7, л.1, м.7

53
Порядок выполнения действий.
п. №
П7, л1,л5, м7

54
Порядок выполнения действий.
п. №
Л.1,л2, п2, м8

55
Степень числа. Квадрат и куб числа
п. №
п.2,м.8.л1,л5

56
Степень числа. Квадрат и куб числа
п. №
П7, л1,л5, м7

57
Умножение и деление натуральных чисел
п. №
П.3, п.2, п.7, л.1, м.7

58
Умножение и деление натуральных чисел
п. №
П7, л1,л5, м7

59
Умножение и деление натуральных чисел
п. №
Л.1,л2, п2, м8

60
Контрольная работа № 5«Деление натуральных чисел»


п.2,м.8.л1,л5

61
Анализ контрольной работы. Как возникла арифметика. Математика чиновников, инженеров и торговцев

творческое задание: Математическое лото.
П7, л1,л5, м7



§4. ПЛОЩАДИ И ОБЪЕМЫ. (14часов)



п. №

62
Формулы
п. 17 № 701, 702
П7, л1,л5, м7

63
Формулы
п. 17 № 704,707
п.3, п.2, п.7, л.1, м.7

64
Площадь. Формула площади прямоугольника
п. 18 № 742,744
п7, л1,л5, м7

65
Площадь. Формула площади прямоугольника
п. 18 № 737,739
п5, л1,л5, м7

66
Единицы измерения площадей
п. 19 №7 79, 780
п.3, п.2, п.7, л.1, м.7

67
Единицы измерения площадей
п. 19 № 782, 785
п7, л1,л5, м7

68

Прямоугольный параллелепипед
п. 20 № 812,814
п5, л1,л5, м7


69
Объемы. Объем прямоугольного параллелепипеда
п. 21 № 840, 841
л.1,л2, п2, м8

70
Объемы. Объем прямоугольного параллелепипеда
п. 21 №842, 846
п5, л1,л5, м7

71
Площади и объемы
п. 22 карточка
п7, л1,л5, м7

72
Площади и объемы
п. 17-21, тест
п.3, п.2, п.7, л.1, м.7

73
Площади и объемы
п. 17-21 карточка
п7, л1,л5, м7

74
Контрольная работа № 6 «Площади и объемы»

л.1,л2, п2, м8

75
Анализ контрольной работы.
Как возникла арифметика. Математика землемеров, архитекторов и строителей
творческое задание: Математическое лото.
л.1,л2, п2, м8


Обыкновенные дроби. (20)


76
Окружность и круг.
п. 22 № 875, 876
п.2,м.8.л1,л5

77
Окружность и круг.
п. 22 № 880
П7, л1,л5, м7

78
Доли. Обыкновенные дроби.
п. 23 № 926,928
л.1,л2, п2, м8

79
Доли. Обыкновенные дроби.
п.23 № 929,930
П7, л1,л5, м7

80
Доли. Обыкновенные дроби.
п. 23 № 934,935

л.1,л2, п2, м8


81
Сравнение дробей.
п. 24, № 966, 967
П7, л1,л5, м7

82
Сравнение дробей.
п.24 № 969, 972
П.3, п.2, п.7, л.1, м.7

83
Правильные и неправильные дроби.
п.25 №999, 1004
П7, л1,л5, м7

84
Правильные и неправильные дроби.
п.25 №1003
Л.1,л2, п2, м8

85
Правильные и неправильные дроби.
п.25 №1004(в,г)
П7, л1,л5, м7

86
Контрольная работа на тему «Сравнение дробей».

п.2,м.8.л1,л5

87
Сложение и вычитание дробей с одинаковыми знаменателями.
п.26 №1041,1044
п.2,м.8.л1,л5

88
Деление и дроби.
п.27 №1076,1077
П.3, п.2, п.7, л.1, м.7

89
Деление и дроби.
п. 27 №1079,1080
П7, л1,л5, м7

90
Смешанные числа.
п.28 №1109, 1111
п.2,м.8.л1,л5

91
Сложение и вычитание смешанных чисел.
п. 29, №1136,1137
П.3, п.2, п.7, л.1, м.7

92
Сложение и вычитание смешанных чисел.
п. 29 №1136, 1138
П7, л1,л5, м7

93
Сложение и вычитание смешанных чисел.
п. 29 №1143, 1141
п.2,м.8.л1,л5

94
Контрольная работа на тему Смешанные числа

п.2,м.8.л1,л5

95
Анализ контрольной работы..Как возникла арифметика. Ломанные числа.
карточка
П.3, п.2, п.7, л.1, м.7


Десятичные дроби.
Сложение и вычитание десятичных дробей (15 ч)


96
Десятичная запись дробных чисел.
п.30 №1166, 1168
П.3, п.2, п.7, л.1, м.7

97
Десятичная запись дробных чисел.
п.30 №1167, 1169
П7, л1,л5, м7


98
Сравнение десятичных дробей.
п.30 №1171
п.2,м.8.л1,л5

99
Сравнение десятичных дробей.
п.31 №1201, 1207
П.3, п.2, п.7, л.1, м.7

100
Сложение и вычитание десятичных дробей.
п.32 №1255,1257
П7, л1,л5, м7

101
Сложение и вычитание десятичных дробей.
п.32 №1256, 1258
п.2,м.8.л1,л5

102
Сложение и вычитание десятичных дробей.
п.32 №1263,1265
П.3, п.2, п.7, л.1, м.7

103
Приближенные значения. Округление чисел.
п. 33 №1297
П7, л1,л5, м7

104
Приближенные значения. Округление чисел.
п. 33 №1324
П.3, п.2, п.7, л.1, м.7

105
Десятичные дроби
карточка
П7, л1,л5, м7

106
Десятичные дроби
тест
Л.1,л2, п2, м8

107
Десятичные дроби
карточка
П7, л1,л5, м7


108
Десятичные дроби
карточка
п.2,м.8.л1,л5

109
Контрольная работа на тему
«Сравнение, сложение и вычитание десятичных дробей».

п.2,м.8.л1,л5

110
Анализ контрольной работы. Как возникла арифметика.
карточка
п.2,м.8.л1,л5


Умножение и деление десятичных дробей. (20ч)


111
Умножение десятичных дробей на натуральное число.
п.34 №1330
п.2,м.8.л1,л5

112
Умножение десятичных дробей на натуральное число.
п.34 №1334, 1335
П7, л1,л5, м7

113
Деление десятичных дробей на натуральные числа
п.35 №1375,
п.2,м.8.л1,л5

114
Деление десятичных дробей на натуральные числа
п.35 №1379
П7, л1,л5, м7

115
Умножение и деление десятичных дробей на натуральные числа
п. 34,35 карточка
п.2,м.8.л1,л5

116
Умножение и деление десятичных дробей на натуральные числа
п. 34,35 тест
П7, л1,л5, м8

117
Умножение и деление десятичных дробей на натуральные числа

п.34,35 карточка
п.2,м.8,л5

118
Контрольная работа № 10

п.2,м.8.л1,л5

119
Умножение десятичных дробей
п. 36 №
П7, л1,л5, м7

120
Умножение десятичных дробей
п. 36 №
л.1,л2, п2, м8

121
Умножение десятичных дробей
п. 36 №
п.2,м.8.л1,л5

122
Деление на десятичную дробь
п.37 №
П7, л1,л5, м8

123
Деление на десятичную дробь
п. 37 №
л.1,л2, п2, м8

124
Среднее арифметическое
п. 38 №
П7, л1,л5, м7

125
Среднее арифметическое
п. 38 №
п.2,м.8.л1,л5

126
Умножение и деление десятичных добей
п.36,37 №
л.1,л2, п2, м8

127
Умножение и деление десятичных добей
п. 36,37 №
П7, л1,л5, м7

128
Умножение и деление десятичных добей
п.36,37 №
П7, л1,л5, м8

129
Умножение и деление десятичных добей
п. 36,37 №
л.1,л2, п2, м8

130
Контрольная работа № 11

п.2,м.8.л1,л5

§8. ИНСТРУМЕНТЫ ДЛЯ ВЫЧИСЛЕНИЙ И ИЗМЕРЕНИЙ. (17 часов)


131
Микрокалькулятор.
п.38 №
п4,л1,м4

132
Микрокалькулятор.
п.38 №
п3,п4, м7,л4

133
Проценты.
п. 39 №
п3, п.2, п.7, л.1, м.7

134
Проценты.

п.39 №
п7, л1,л5, м7

135
Проценты.

п. 39 №
л.1,л2, п2, м8

136
Проценты.

п. 39 №
п7, л1,л5, м7

137
Проценты.

п.39
л.1,л2, п2, м8

138
Контрольная работа на тему: Микрокалькулятор. Проценты.


п.2,м.8.л1,л5

139
Угол. Прямой и развернутый угол. Чертежный треугольник
п. 40 №
П.3, п.2, п.7, л.1, м.7

140
Угол. Прямой и развернутый угол. Чертежный треугольник
п.40,41 №
П7, л1,л5, м7

141
Измерение углов. Транспортир
п. 42 №
Л.1,л2, п2, м8

142
Измерение углов. Транспортир
п. 42 №


143
Круговые диаграммы
п.43 №
П.3, п.2, п.7, л.1, м.7

144
Круговые диаграммы
п. 43 №
П7, л1,л5, м7

145
Измерения и вычисления
п.40-43 карточка
Л.1,л2, п2, м8

146
Измерения и вычисления
п40-43 карточка
л.1,л2, п2, м8

147
Контрольная работа на тему: Чертежные инструменты и измерения.


п.2,м.8.л1,л5


Описательная статистика. Вероятность. Комбинаторика 5 ч.


148
Введение в вероятность. Перебор возможных вариантов
п. №
П.3, п.2, п.7, л.1, м.7

149
Дерево возможных вариантов
п. №
П7, л1,л5, м7

150
Достоверные, невозможные и случайные события
п. №
Л.1,л2, п2, м8

152
Решение простейших комбинаторных задач
п. №
П.3, п.2, п.7, л.1, м.7

153
Обобщающий урок по теме «Вероятность»

п. №
П7, л1,л5, м7

Итоговое повторение, демонстрация личных достижений учащихся 20 ч


п. №
Л.1,л2, п2, м8

154
Арифметические действия с натуральными и дробными числами
п. №
П.3, п.2, п.7, л.1, м.7

155
Арифметические действия с натуральными и дробными числами
п. №
П7, л1,л5, м7

156
Арифметические действия с натуральными и дробными числами
п. №
Л.1,л2, п2, м8

157
Буквенные выражения, упрощение выражений, формулы
п. №
П.3, п.2, п.7, л.1, м.7

158
Буквенные выражения, упрощение выражений, формулы
п. №
П7, л1,л5, м7

159
Буквенные выражения, упрощение выражений, формулы
п. №
Л.1,л2, п2, м8

160
Уравнения, решение задач с помощью уравнений
п. №
П.3, п.2, п.7, л.1, м.8

161
Уравнения, решение задач с помощью уравнений
п. №
П7, л1,л5, м7

162
Уравнения, решение задач с помощью уравнений
п. №
Л.1,л2, п2, м8

163
Измерения и вычисления. Проценты. Круговые диаграммы
п. №
П.3, п.2, п.7, л.1, м.7

164
Измерения и вычисления. Проценты. Круговые диаграммы
п. №
П7, л1,л5, м7

165
Измерения и вычисления. Проценты. Круговые диаграммы
п. №
Л.1,л2, п2, м8

166
Измерения и вычисления. Проценты. Круговые диаграммы
п. №
П.3, п.2, п.7, л.1, м.7

167
Наглядная геометрия
п. №
П7, л1,л5, м7

168
Наглядная геометрия
п. №
Л.1,л2, п2, м8

169
Итоговая контрольная работа
п. №
п.2,м.8.л1,л5

170
Наглядная геометрия
п. №
П.3, п.2, п.7, л.1, м.7

171
Мир еще не рассказан, можешь его вдохнуть. Разве тебе заказан к тайнам великий путь?
п. №
П7, л1,л5, м7

172

Мир еще не рассказан, можешь его вдохнуть. Разве тебе заказан к тайнам великий путь?
творческое задание- составление кроссворда по пройденной теме
Л.1,л2, п2, м8

173
Мир еще не рассказан, можешь его вдохнуть. Разве тебе заказан к тайнам великий путь?
Творческое задание: математическое лото.
П.3, п.2, п.7, л.1, м.7

174

Мир еще не рассказан, можешь его вдохнуть. Разве тебе заказан к тайнам великий путь?
творческое задание- составление кроссворда
П7, л1,л5, м7

175
Мир еще не рассказан, можешь его вдохнуть. Разве тебе заказан к тайнам великий путь?
п. №
Л.1,л2, п2, м8



VIII. Учебно-методические средства обучения.

Основная литература.
В.И. Жохов. Преподавание математики в 5 и 6 классах. Метод. рекомендации для учителя к учебникам Н.Я. Виленкина и др. М.: Мнемозина,



2. Виленкин Н.Я. Математика. 5 кл. – М.; Мнемозина, 2012.
3. Жохов В.И., Крайнева Л.Б. Контрольные работы. 5 кл. - М.; Мнемозина, 2012
Жохов В.И., Митяева И.М., Математические диктанты. 5 кл. - М.; Мнемозина, 2012
Жохов В.И., Погодин. В.Н. Математический тренажер 5, 6 кл. - М.; Мнемозина,20 12
Ершова А.П., Голобородько В.В. Самостоятельные и контрольные работы 5 кл.- М.: ИЛЕКСА, 2010
Рудницкая В.Н. Математика. 5 кл. - Рабочая тетрадь №1: Учебное пособие для общеобразоват. учреждений.- М.; Мнемозина, 2010
Рудницкая В.Н. Математика. 5 кл.: Рабочая тетрадь №2: Учебное пособие для общеобразоват. учреждений.- М.; Мнемозина, 2010
Чесноков А.С., Нешков К.И. Дидактические материалы по математике 5 класс. – М.: Просвещение, 2012

Дополнительная литература.
Депман И.Я., Виленкин Н.Я. За страницами учебника математики.- М.: Просвещение, 2001
Лебединцева Е.А., Беленкова Е.Ю. Математика. Задания для обучения и развития учащихся. – М.: Интеллект-Центр, 2009
Попова Л.П. Поурочные разработки по математике к учебному комплекту Н.Я.Виленкина 5 класс. – М.: ВАКО, 2009 г.
Шарыгин И.Ф., Шевкин А.В. Математика. Задачи на смекалку.- М.: Просвещение, 2012
Шарыгин И.Ф. Наглядная геометрия. – М.: МИРОС, 2012
Шевкин А.В. Сборник задач по математике для 5-6 классов. – М.: Русское слово,2001













Интернет ресурсы:
Газета «Математика» «издательского дома» «Первое сентября» http://[ Cкачайте файл, чтобы посмотреть ссылку ]. 1[ Cкачайте файл, чтобы посмотреть ссылку ]
Материалы по математике в Единой коллекции цифровых образовательных ресурсов
[ Cкачайте файл, чтобы посмотреть ссылку ] ; http://www.[ Cкачайте файл, чтобы посмотреть ссылку ] 
Общероссийский математический портал [ Cкачайте файл, чтобы посмотреть ссылку ][ Cкачайте файл, чтобы посмотреть ссылку ]
Портал [ Cкачайте файл, чтобы посмотреть ссылку ] - вся математика в одном месте
http ://www. [ Cкачайте файл, чтобы посмотреть ссылку ] 
Фестиваль педагогических идей «Открытый урок» [ Cкачайте файл, чтобы посмотреть ссылку ]





ЦИФРОВЫЕ ОБРАЗОВАТЕЛЬНЫЕ РЕСУРСЫ,
используемые на уроках математики в 5 классе.
Использование на уроках математики в 5 классе цифровых образовательных ресурсов Единой коллекции ЦОР является целесообразным, во-первых, потому, что является электронным компонентом учебно-методического комплекса по математике под редакцией Н.Я.Виленкина. Во-вторых, ЦОРы поддерживают все этапы работы с учебным материалом, т. е. могут использоваться при объяснении, тренировке и контроле; они предназначены как для коллективной, так и для индивидуальной учебной деятельности. В-третьих, учителю не надо порой «изобретать велосипед»: включение в систему работы на уроке цифровых образовательных ресурсов единой коллекции делает урок насыщенней по способам обучения и результативным по усвоению содержания
№ урока
Тема урока
Назначение ЦОР
ИКТ



Обозначение натуральных чисел.
демонстрационно-опорные материалы
тренировочно-контролирующий материал
Чтение и запись больших натуральных чисел
[ Cкачайте файл, чтобы посмотреть ссылку ]

Математический диктант по теме
[ Cкачайте файл, чтобы посмотреть ссылку ]



Отрезок. Длина отрезка. Треугольник.
учебно-справочный материал
Электронное пособие "Отрезок. Длина отрезка."
13 LI
·
·
·
·
·
·
·
·
·
·
·
·
·
·
·
·
·
·
·
·
·
·
·
·
·
·
·
·
·
·
·
·
·
·
·
·
·
·
·
·
·
·
·
·
·
·
·
·
·
·
·
·
·
·
·
·
·
·
·
·
·
·
·
·
·
·
·
·
·
·
·
·
·
·
·
·
·
·
·
·
·
·
·
·
·
·
·
·
·
·
·
·
·
·
·
·
·
·
·
·
·
·
·
·
·
·
·
·
·
·
·
·
·
·
·
·
·
·
·
·
·
·
·
·
·
·
·Плоскость, прямая, луч.
учебно-справочный материал
Лекция «Прямая, луч».
[ Cкачайте файл, чтобы посмотреть ссылку ]



Шкалы и координаты.
Фронт. работа на этапе введения новых знаний
тренировочно-контролирующий материал
Задача на движение вдогонку. Координатный луч.
[ Cкачайте файл, чтобы посмотреть ссылку ]

Координатный луч.
[ Cкачайте файл, чтобы посмотреть ссылку ]



Меньше или больше.




Сложение натуральных чисел и его свойства.
Учебно-справочный материал
Контроль и коррекция
Переместительный и сочетательный законы сложения
[ Cкачайте файл, чтобы посмотреть ссылку ]

Переместительный закон сложения
[ Cкачайте файл, чтобы посмотреть ссылку ]



Вычитание.




Числовые и буквенные выражения.
учебно-справочный материал
тренировочно-контролирующий материал
Иллюстрация «Буквенное выражение и его значение.»
[ Cкачайте файл, чтобы посмотреть ссылку ]

5 заданий на усвоение материала параграфа
[ Cкачайте файл, чтобы посмотреть ссылку ]
Задание повышенной сложности на усвоение материала
[ Cкачайте файл, чтобы посмотреть ссылку ]



Буквенная запись свойств сложения и вычитания
Фронт. работа на этапе введения новых знаний
Первичный контроль и коррекция знаний
Контроль и коррекция знаний
Переместительный и сочетательный законы сложения
[ Cкачайте файл, чтобы посмотреть ссылку ]

Игровое задание. Переместительный закон сложения.
[ Cкачайте файл, чтобы посмотреть ссылку ]

Законы арифметических действий
[ Cкачайте файл, чтобы посмотреть ссылку ]


Уравнение
Фронт. работа на этапе введения
Демонстрационно-опорные материалы
Контроль и коррекция знаний
Обобщающий материал

Демонстрационно-опорные материалы

Решение простейших уравнений
[ Cкачайте файл, чтобы посмотреть ссылку ]
Уравнения. Решение уравнений.
[ Cкачайте файл, чтобы посмотреть ссылку ]


Уравнения
[ Cкачайте файл, чтобы посмотреть ссылку ]
Уравнения. Решение уравнений.
[ Cкачайте файл, чтобы посмотреть ссылку ]

Решение задач с помощью уравнений
[ Cкачайте файл, чтобы посмотреть ссылку ]
Алгоритм решения задач
[ Cкачайте файл, чтобы посмотреть ссылку ]



Умножение натуральных чисел и его свойства.
Фронт. работа на этапе введения новых знаний
Распределительный закон умножения
[ Cкачайте файл, чтобы посмотреть ссылку ]



Деление.




Деление с остатком.




Упрощение выражений.
Игровое задание . Устные приемы счета
Демонстрационный материал
Первичный контроль с самопроверкой

Замена двух действий одним.
[ Cкачайте файл, чтобы посмотреть ссылку ]

Использование распределительного закона
[ Cкачайте файл, чтобы посмотреть ссылку ]
Использование распределительного закона. Упрощение выраж.
[ Cкачайте файл, чтобы посмотреть ссылку ]



Степень числа. Квадрат и куб числа.
Демонстрационно-опорные материалы
Т
Тренировочно-контролирующий материал
Квадрат числа.
[ Cкачайте файл, чтобы посмотреть ссылку ]

Вычисление квадратов и кубов чисел. Порядок действий.
[ Cкачайте файл, чтобы посмотреть ссылку ]

Найти значение ввыражения
[ Cкачайте файл, чтобы посмотреть ссылку ]



Формулы.
Фронт. работа на этапе введения новых знаний

Контроль и коррекция знаний
Понятие формулы. Формулы площади и периметра прямоугольника. Формула пути
[ Cкачайте файл, чтобы посмотреть ссылку ]

Формулы пути и площади
[ Cкачайте файл, чтобы посмотреть ссылку ]


Площадь. Формула площади прямоугольника.
Фронт. работа на этапе введения новых знаний
Контроль и коррекция знаний
Прямоугольник, его свойства, площадь и периметр.
[ Cкачайте файл, чтобы посмотреть ссылку ]

Площадь и периметр прямоугольника.
[ Cкачайте файл, чтобы посмотреть ссылку ]


Единицы измерения площадей.
Демонстрационно-опорные материалы
Тренировочно-контролирующий материал

Переход от одних единиц измерения к другим
[ Cкачайте файл, чтобы посмотреть ссылку ]

Переход от одних единиц измерения к другим
[ Cкачайте файл, чтобы посмотреть ссылку ]


Прямоугольный параллелепипед.
Демонстрационный материал



Тренировочно-контролирующий материал
Прямоугольный параллелепипед, опред., свойства.
[ Cкачайте файл, чтобы посмотреть ссылку ]
[ Cкачайте файл, чтобы посмотреть ссылку ]
Куб
[ Cкачайте файл, чтобы посмотреть ссылку ]

Куб. Прямоугольный параллелепипед. 5 задач
[ Cкачайте файл, чтобы посмотреть ссылку ]



Объемы. Объем прямоугольного параллелепипеда.
Демонстрационный материал





Тренировочно-контролирующий материал
Объем.
[ Cкачайте файл, чтобы посмотреть ссылку ]
Измерение объема.
[ Cкачайте файл, чтобы посмотреть ссылку ]
Объем куба.
[ Cкачайте файл, чтобы посмотреть ссылку ]
Переход от одних единиц измерения объема к другим.
[ Cкачайте файл, чтобы посмотреть ссылку ]

Задачи на вычисление объема и одного из измерений
[ Cкачайте файл, чтобы посмотреть ссылку ]
[ Cкачайте файл, чтобы посмотреть ссылку ]
[ Cкачайте файл, чтобы посмотреть ссылку ]



Окружность и круг.
Демонстрационный материал


Тренировочный материал

Окружность. Радиус. Хорда.
[ Cкачайте файл, чтобы посмотреть ссылку ]
Понятие круга.
[ Cкачайте файл, чтобы посмотреть ссылку ]

Окружность.
[ Cкачайте файл, чтобы посмотреть ссылку ]
Окружность, хорда, радиус.
[ Cкачайте файл, чтобы посмотреть ссылку ]
Круг.
[ Cкачайте файл, чтобы посмотреть ссылку ]



Доли. Обыкновенные дроби.
Демонстрационный материал
Фронт. работа на этапе введения новых знаний
тренировочно-контролирующий материал
Обыкновенные дроби.
[ Cкачайте файл, чтобы посмотреть ссылку ]
Дробь как одна или несколько равных долей
[ Cкачайте файл, чтобы посмотреть ссылку ]

Математический диктант. Обыкновенные дроби
[ Cкачайте файл, чтобы посмотреть ссылку ]



Сравнение дробей.
тренировочно-контролирующий материал
Сравнение чисел и дробей
[ Cкачайте файл, чтобы посмотреть ссылку ]
[ Cкачайте файл, чтобы посмотреть ссылку ]



Правильные и неправильные дроби.
Демонстрационный материал

Фронт. работа на этапе введения новых знаний

Первичный контроль и коррекция
Тренировочно-контролирующий материал
Правильные дроби
[ Cкачайте файл, чтобы посмотреть ссылку ]
Неправильные дроби
[ Cкачайте файл, чтобы посмотреть ссылку ]
Понятия: правильная, неправильная дробь, смешанное число. [ Cкачайте файл, чтобы посмотреть ссылку ]

Выделение целой части неправильной дроби.
[ Cкачайте файл, чтобы посмотреть ссылку ]

Математический диктант.
[ Cкачайте файл, чтобы посмотреть ссылку ]



Сложение и вычитание дробей с одинаковыми знаменателями.
Фронт. работа на этапе введения новых знаний
Тренировочные упражнения
тренировочно-контролирующий материал
Сложение и вычитание дробей с одинаковыми знаменателями [ Cкачайте файл, чтобы посмотреть ссылку ]

Игровое задание
[ Cкачайте файл, чтобы посмотреть ссылку ]
Вычитание из единицы обыкновенной дроби
[ Cкачайте файл, чтобы посмотреть ссылку ]
Самостоятельная работа
[ Cкачайте файл, чтобы посмотреть ссылку ]



Деление и дроби.
Демонстрация


Тренировочно-контролирующий материал
Изучение нового
[ Cкачайте файл, чтобы посмотреть ссылку ]
[ Cкачайте файл, чтобы посмотреть ссылку ]

Опрос
[ Cкачайте файл, чтобы посмотреть ссылку ]



Смешанные числа.
Новый материал



Контроль и коррекция
Понятие смешанного числа
[ Cкачайте файл, чтобы посмотреть ссылку ]
Выделение целой части неправильной дроби
[ Cкачайте файл, чтобы посмотреть ссылку ]

Самостоятельная работа
[ Cкачайте файл, чтобы посмотреть ссылку ]
[ Cкачайте файл, чтобы посмотреть ссылку ]



Сложение и вычитание смешанных чисел.

Новый материал

демонстрационно-опорные материалы
Контролирующие материалы
Сложение
[ Cкачайте файл, чтобы посмотреть ссылку ]
Вычитание (случай1)
[ Cкачайте файл, чтобы посмотреть ссылку ]
(случай 2) [ Cкачайте файл, чтобы посмотреть ссылку ]
(случай 3) [ Cкачайте файл, чтобы посмотреть ссылку ]
Операции над смешанными числами
[ Cкачайте файл, чтобы посмотреть ссылку ]



Десятичная запись дробных чисел.
Новый материал



Контроль и коррекция
Тренировочно-контролирующий материал
Конспект (понятие десятичной дроби)
[ Cкачайте файл, чтобы посмотреть ссылку ]
(чтение и запись)
[ Cкачайте файл, чтобы посмотреть ссылку ]

Математический диктант
[ Cкачайте файл, чтобы посмотреть ссылку ]

Задание на усвоение материала параграфа
[ Cкачайте файл, чтобы посмотреть ссылку ]
Задания повышенной сложности
[ Cкачайте файл, чтобы посмотреть ссылку ]



Сравнение десятичных дробей.

Новый материал

Контролирующий материал
Лекция по теме
[ Cкачайте файл, чтобы посмотреть ссылку ]
[ Cкачайте файл, чтобы посмотреть ссылку ]

Задания на усвоение материала параграфа
[ Cкачайте файл, чтобы посмотреть ссылку ]
Задания повышенной сложности
[ Cкачайте файл, чтобы посмотреть ссылку ]



Сложение и вычитание десятичных дробей.
Новый материал

Контроль и коррекция

Контролирующие материалы
Лекция по теме
[ Cкачайте файл, чтобы посмотреть ссылку ]

Диктант по теме
[ Cкачайте файл, чтобы посмотреть ссылку ]

Задания на усвоение материала параграфа
[ Cкачайте файл, чтобы посмотреть ссылку ]
Задания повышенной сложности
[ Cкачайте файл, чтобы посмотреть ссылку ]



Приближенные значения чисел. Округление чисел.
Новый материал

Тренировочно-контролирующий материал
Лекция по теме
[ Cкачайте файл, чтобы посмотреть ссылку ]

Диктант по теме
[ Cкачайте файл, чтобы посмотреть ссылку ]
Задания на усвоение материала параграфа
[ Cкачайте файл, чтобы посмотреть ссылку ]
Задания повышенной сложности
[ Cкачайте файл, чтобы посмотреть ссылку ]



Умножение десятичных дробей на натуральные числа.
тренировочно-контролирующий материал
Игровое задание
[ Cкачайте файл, чтобы посмотреть ссылку ]


Деление десятичных дробей на натуральные числа.
Новый материал

Контроль и коррекция


Лекция по теме
[ Cкачайте файл, чтобы посмотреть ссылку ]

Игровое задание
[ Cкачайте файл, чтобы посмотреть ссылку ]

Диктант
[ Cкачайте файл, чтобы посмотреть ссылку ]
Задания на усвоение материала параграфа
[ Cкачайте файл, чтобы посмотреть ссылку ]
Задания повышенной сложности
[ Cкачайте файл, чтобы посмотреть ссылку ]



Умножение десятичных дробей.
Новый материал



Тренировочные упражнения


Контролирующие материалы
Лекция
[ Cкачайте файл, чтобы посмотреть ссылку ]
Алгоритм
[ Cкачайте файл, чтобы посмотреть ссылку ]

Текстовые задачи на умножение десятичных дробей
[ Cкачайте файл, чтобы посмотреть ссылку ]
[ Cкачайте файл, чтобы посмотреть ссылку ]
[ Cкачайте файл, чтобы посмотреть ссылку ]

Задания на усвоение материала параграфа
[ Cкачайте файл, чтобы посмотреть ссылку ]
Задания повышенной сложности
[ Cкачайте файл, чтобы посмотреть ссылку ]



Деление на десятичную дробь.
Новый материал

Тренировочно-контролирующий материал
Конспект по теме
[ Cкачайте файл, чтобы посмотреть ссылку ]

Текстовые задачи
[ Cкачайте файл, чтобы посмотреть ссылку ]
[ Cкачайте файл, чтобы посмотреть ссылку ]

Задания на усвоение материала параграфа
[ Cкачайте файл, чтобы посмотреть ссылку ]
Задания повышенной сложности
[ Cкачайте файл, чтобы посмотреть ссылку ]



Среднее арифметическое.
демонстрационно-опорные материалы
тренировочно-контролирующий материал

Фронтальная работа на этапе усвоения нового
[ Cкачайте файл, чтобы посмотреть ссылку ]

Текстовая задача
[ Cкачайте файл, чтобы посмотреть ссылку ]
[ Cкачайте файл, чтобы посмотреть ссылку ]



Микрокалькулятор.
тренировочно-контролирующий материал
Вычисление процента от числа с помощью МК
[ Cкачайте файл, чтобы посмотреть ссылку ]
Сложение и вычитание с помощью МК
[ Cкачайте файл, чтобы посмотреть ссылку ]
Умножение и деление с помощью МК
[ Cкачайте файл, чтобы посмотреть ссылку ]



Проценты.
Фронтальная работа на этапе введения новых знаний
демонстрационно-опорные материалы

тренировочный материал

контролирующий материал
Введение понятия процента
[ Cкачайте файл, чтобы посмотреть ссылку ]
[ Cкачайте файл, чтобы посмотреть ссылку ]
Нахождение процента от числа
[ Cкачайте файл, чтобы посмотреть ссылку ]
Обратная задача
[ Cкачайте файл, чтобы посмотреть ссылку ]

Простейшие задачи на проценты
[ Cкачайте файл, чтобы посмотреть ссылку ]
Самостоятельная работа
[ Cкачайте файл, чтобы посмотреть ссылку ]
[ Cкачайте файл, чтобы посмотреть ссылку ]
[ Cкачайте файл, чтобы посмотреть ссылку ]



Угол. Прямой и развернутый угол. Чертежный треугольник.
Демонстрационно-опорные материалы
Контроль и коррекция

Определение угла; развернутого угла.
[ Cкачайте файл, чтобы посмотреть ссылку ]

Математический диктант «Углы»
[ Cкачайте файл, чтобы посмотреть ссылку ]



Измерение углов. Транспортир.
Демонстрационно-опорные материалы



Контроль и коррекция
Острый, тупой угол. [ Cкачайте файл, чтобы посмотреть ссылку ]
Транспортир. Измерение углов с помощью транспортира.
[ Cкачайте файл, чтобы посмотреть ссылку ]
Биссектриса угла.
[ Cкачайте файл, чтобы посмотреть ссылку ]

Математический диктант «Измерение углов»
[ Cкачайте файл, чтобы посмотреть ссылку ]



Круговые диаграммы.



Х. Контрольные материалы

К-1. Натуральные числа и шкалы.

Вариант 1
Вариант 2

№1.Запишите цифрами числа:
а) восемь миллионов две тысячи три; б) три миллиарда пятьсот тысяч.
№2. На луче AD постройте отрезки AB и BC так, чтобы
AB = 2 см 5 мм и BC = 6 см 5 мм
Чему равна длина отрезка AC?
№3. Начертите прямую AB,луч CD и треугольник MNK так, чтобы
а) луч CD пересекал прямую AB; б) отрезок MN пересекал прямую AB; в)отрезок NK не пересекал прямую AB.
Пересекает ли отрезок MK прямую AB?
№4. Определите, каким числам соответствуют точки A, B, C и D на рисунке:

№5. Запишите наименьшее семизначное число, все цифры которого различны.
№1.Запишите цифрами числа:
а) десять миллионов сто тысяч пять; б) семь миллиардов две тысячи.
№2. На луче AD постройте отрезки AB и BC так, чтобы
AB = 3 см и BC = 4 см 5 мм
Чему равна длина отрезка AC?
№3. Начертите прямую AB,луч CD и треугольник MNK так, чтобы
а) луч CD не пересекал прямую AB; б) отрезок MN не пересекал прямую AB; в)отрезок NK пересекал прямую AB.
Пересекает ли отрезок MK прямую AB?
№4. Определите, каким числам соответствуют точки A, B, C и D на рисунке:



№5.Запишите наибольшее семизначное число, все цифры которого различны.


К – 2. Сложение и вычитание натуральных чисел.

Вариант 1
Вариант 2

№1. Выполните действия:

№2. Сравните значения выражений

№3.Одна сторона треугольника равна 28 см, вторая на 8 см меньше первой, а третья – на 2 см меньше второй. Найдите периметр треугольника.
№4.В автобусе было 98 пассажиров. На первой остановке вышло 15 и вошло 27 пассажиров, на второй остановке вышло 24 и вошло 18 пассажиров. Сколько пассажиров стало в автобусе после второй остановки?
№5. Найдите число, которое при увеличении втрое увеличится на 24.
№1. Выполните действия:

№2. Сравните значения выражений

№3. Одна сторона треугольника равна 34 см, вторая на 2 см больше первой, а третья – на 4 см меньше первой. Найдите периметр треугольника.
№4.В автобусе было 98 пассажиров. На первой остановке вышло 27 и вошло 14 пассажиров, на второй остановке вышло 17 и вошло 25 пассажиров. Сколько пассажиров стало в автобусе после второй остановки?
№5. Найдите число, которое при уменьшении на 24 уменьшится вдвое.




К – 3. Буквенные выражения.

Вариант 1
Вариант 2

№1. Упростите выражение и найдите его значение:

№2. Решите уравнения:
а) 568 – x = 436; б) (y + 47) – 17 = 39.
№3. Составьте буквенное выражение для решения задачи.
Длина ломаной ABCD равна 68 см. Звено AB равно 16 см, а звено BC на a см длиннее звена AB. Найдите длину звена CD.
Упростите выражение и вычислите при a = 24.
№4. Задумали некоторое число. Из 167 вычли задуманное число, из полученной разности вычли 53 и получили 29. Какое число было задумано?
№5. Каким числом необходимо заменить a, чтобы корнем уравнения x + 6 = a было число 12?
№1. Упростите выражение и найдите его значение:

№2. Решите уравнения:
а) x – 461 = 253; б) 73 – (13 + y) = 21.
№3. Составьте буквенное выражение для решения задачи.
Длина ломаной KLMN равна 73 см. Звено KL равно 18 см, а звено LM на bсм длиннее звена KL. Найдите длину звена MN.
Упростите выражение и вычислите при b = 12.
№4. Задумали некоторое число. К 125 прибавили задуманное число, из полу- ченной суммы вычли 39 и получили 87. Какое число было задумано?
№5. Каким числом необходимо заменить a, чтобы корнем уравнения x - a = 6 было число 12?



К – 4. Умножение и деление натуральных чисел.

Вариант 1
Вариант 2

№1. Вычислите:

№2. Упростите выражение 13 QUOTE 1415.
№3. Решите уравнения:

№4. На складе было 783 т цемента. Ежедневно с него вывозили по 27 т, после чего осталось 351 т цемента. Сколько дней со склада вывозили цемент?
№5.Как изменится частное, если делимое уменьшить в 2 раза, а делитель – в 4 раза?

№1. Вычислите:

№2. Упрстите выражение 13 QUOTE 1415.
№3. Решите уравнения:

№4. На овощной базе было 927 кг картофеля. В течение 16 дней с нее забирали по одному мешку картофеля, после чего осталось 159 кг. Какова масса мешка картофеля?
№5. Как изменится частное, если делимое увеличить в 2 раза, а делитель – в 6 раз?





К – 5. Все действия с натуральными числами.

Вариант 1
Вариант 2

№1.Найдите значение выражения:
а) 13 QUOTE 1415 б) 13 QUOTE 1415.
№2. Упростите выражение 31a + 127 + 48a. Вычислите при a = 4 ; 10.
№3. Решите уравнения:

№4. Слесарь и его ученик изготовили 192 детали. Слесарь работает в 3 раза быстрее ученика. Сколько деталей сделал каждый из них?
№5. Угадайте корень уравнения 13 QUOTE 1415

№1.Найдите значение выражения:
а) 13 QUOTE 1415 б) 13 QUOTE 1415.
№2. Упростите выражение 23m + 174 + 6m. Вычислите при m = 8 ; 100
№3. Решите уравнения:

№4. Две швеи сшили 183 костюма, причем одна из них работает в 2 раза быстрее другой. Сколько костюмов сшила каждая швея?
№5. Угадайте корень уравнения 13 QUOTE 1415


К – 6. Площади и объемы.

Вариант 1
Вариант 2

№1.Числа x и y связаны формулой 13 QUOTE 1415. Найдите:
а) y, если x = 29; б) x, если y = 231.
№2. Выразите:
а) в квадратных метрах: 19 а; 25 га; 3 а 59 м2;
б) в арах: 38 га; 3 га 12 а; 3200 м2.
№3. Длина прямоугольной грядки равна 3 м 6дм, а ширина на 1 м 8 дм меньше длины. Найдите периметр и площадь грядки.
№4. Комната имеет форму прямоугольного параллелепипеда. Ее объем равен 72 м2, а высота – 3 м. Найдите площадь потолка этой комнаты.
№5. Составьте формулу для нахождения длины прямоугольного параллелепипеда а, если известна его ширина b, высота h и объем V.

№1.Числа x и y связаны формулой 13 QUOTE 1415. Найдите:
а) y, если x = 17; б) x, если y = 303.
№2. Выразите:
а) в квадратных метрах: 27 а; 65 га; 4 а 39 м2;
б) в арах: 76 га; 4 га 25 а; 4800 м2.
№3. Ширина прямоугольного коридора равна 1 м 8 дм, а длина на 3 м 6 дм больше ширины. Найдите периметр и площадь коридора.
№4. Спортзал имеет форму прямоугольного параллелепипеда. Его объем равен 432 м3, а высота – 4 м. Найдите площадь пола этого спортзала.
№5. Составьте формулу для нахождения ширины прямоугольного параллелепипеда b, если известна его длина а, высота h и объем V.






К – 7. Натуральные числа.

Вариант 1
Вариант2

№1. Выполните действия:
13 QUOTE 1415:15.
№2. Решите уравнения: а) 33x – 17x + 14 = 558;
б) 988 : (7x - 18) = 26.
№3.На соревнованиях по метанию мяча Ваня бросил мяч в три раза дальше, чем Таня. На сколько метров метнули мяч Ваня и Таня, если Ваня бросил мяч на 24 м дальше, чем Таня?
№4.Прямоугольный лист, длина которого 36 см, а ширина на 12 см меньше длины, разрезали на прямоугольные пластинки размером 6Ч9 см. Сколько всего пластинок получилось?
№5. При каком наибольшем натуральном значении x выполняется неравенство 111 – x > 98?
№1. Выполните действия:
13 QUOTE 1415:25.
№2. Решите уравнения: а) 42x + 19x - 16 = 1448;
б) 936 : (8x - 17) = 24.
№3. На новогодней елке шаров в 5 раз больше, чем всех остальных игрушек. Сколько всего игрушек украшают новогоднюю елку, если шаров на 76 больше, чем остальных игрушек?
№4. Прямоугольное стекло, ширина которого 18 см, а длина на 30 см больше ширины, разрезали на прямоугольные стекла размером 6Ч9 см. Сколько таких стекол получилось?
№5. При каком наименьшем натуральном значении x выполняется неравенство 105 – x < 93?


К – 8. Обыкновенные дроби.

Вариант 1
Вариант 2

№1. Выразите в метрах: а) 3 дм; б) 32 см; в) 2 мм.
№2. Постройте отрезок AB = 8 см. Отметьте на этом отрезке точки C, D и E так, чтобы


№3. Расположите в порядке возрастания дроби:

№4. Папе 40 лет. Возраст Володи составляет 13 QUOTE 1415 возраста папы и 13 QUOTE 1415 возраста мамы. Сколько лет маме и сколько – Володе?
№5. При каком x верно равенство

№1. Выразите в метрах: а) 6 дм; б) 17 см; в) 7 мм.
№2. Постройте отрезок AB = 8 см. Отметьте на этом отрезке точки C, D и E так, чтобы


№3. Расположите в порядке убывания дроби:

№4. Тыква весит 24 кг. Масса арбуза составляет 13 QUOTE 1415 массы тыквы и 13 QUOTE 1415 массы дыни. Какова масса арбуза и какова масса дыни?
№5. При каком x верно равенство






К – 9. Сложение и вычитание обыкновенных дробей и смешанных чисел.

Вариант 1
Вариант 2

№1. Выполните действия:

№2. Решите уравнения:

№3. Рома выполнял домашние задания 3ч, из них математику – 113 QUOTE 1415ч, а родной язык - 13 QUOTE 1415ч. Сколько времени затратил Рома на остальные предметы?
№4. 8 л молока разлили в 5 одинаковых банок. Сколько литров молока в каждой банке?
№5. Какое наименьшее натуральное число удовлетворяет неравенству 13 QUOTE 1415
№1. Выполните действия:

№2. Решите уравнения:

№3. Из 6 соток дачного участка 113 QUOTE 1415 сотки занимают постройки, 213 QUOTE 1415 сотки – грядки, а остальную площадь – газон. Сколько соток занимает газон?
№4. 7 кг масла разрезали на 4 одинаковых куска. Какова масса каждого куска масла?
№5. . Какое наименьшее натуральное число удовлетворяет неравенству 13 QUOTE 1415


К – 10. Сложение и вычитание десятичных дробей.

Вариант 1
Вариант 2

№1. Выполните действия:

№2. Решите уравнения:

№3. Собственная скорость лодки 8,7 км/ч, а скорость течения – 2,9 км/ч. Найдите скорость лодки по течению и против течения.
№4. Округлите каждое из чисел а) 584,356; б) 935,0846; в) 0,8355 до единиц, до десятых и до сотых.
№5. Замените звездочку цифрой так, чтобы получилось верное неравенство: 13 QUOTE 1415
№1. Выполните действия:

№2. Решите уравнения:

№3. Собственная скорость байдарки равна 8,9 км/ч, а скорость течения – 3,2 км/ч. Найдите скорость лодки по течению и против течения.
№4. Округлите каждое из чисел а) 573,856; б) 846,0739; в) 0,325 до единиц, до десятых и до сотых.
№5. Замените звездочку цифрой так, чтобы получилось верное неравенство: 13 QUOTE 1415






К – 11. Умножение и деление десятичных дробей на натуральные числа.


Вариант 1
Вариант 2

№1. Вычислите:

№2. Выполните действия: 8,2 – 0,96 : 8
· 60.
№3. Решите уравнения:

№4. 5 машин песка и 3 машины щебня имеют массу 22,8 т. Найдите массу одной машины щебня, если масса одной машины песка равна 2,7 т.
№5. Сумма двух чисел равна 16,2, а разность – 2,6. Найдите эти числа.
№1. Вычислите:

№2. Выполните действия: 7,6 – 0,98 : 7
· 40.
№3. Решите уравнения:

№4. 4 пакета картофеля и 3 пакета моркови имеют массу 15,8 кг. Найдите массу одного пакета моркови, если масса одного пакета картофеля равна 2,6 кг.
№5. Сумма двух чисел равна 14,6, а разность – 2,8. Найдите эти числа.



К – 12. Все действия с десятичными дробями.

Вариант 1
Вариант 2

№1. Выполните действия:

№2. Вычислите:

№3. Решите уравнения:

№4. Из города одновременно в противо-положных направлениях выехали два автомобиля. Скорость первого автомобиля равна 82,4 км/ч, а скорость второго – 76,3 км/ч. Через сколько часов расстояние между ними будет равно 126,96 км?
№5. Запишите формулу среднего арифметического двух чисел, одно из которых в 3 раза больше другого.

№1. Выполните действия:

№2. Вычислите:

№3. Решите уравнения:

№4. Из двух пунктов, расстояние между которыми равно 34,68 км, одновременно навстречу друг другу вышли два пешехода. Скорость первого пешехода равна 4,8 км/ч, а скорость второго – 5,4 км/ч. Через сколько часов пешеходы встретятся?
№5. Запишите формулу среднего арифметического двух чисел, одно из которых в 5 раз меньше другого.










К – 13. Проценты.

Вариант 1
Вариант 2

№1.Площадь поля 260 га. Горохом засеяно 35% поля. Какую площадь занимают посевы гороха?
№2.Найдите значение выражения
201 – (176,4 : 16,8 + 9,68) 13 QUOTE 1415 2,5.
№3.В библиотеке 12% всех книг – словари. Сколько книг в библиотеке, если словарей в ней 900?
№4. Решите уравнение 12 + 8,3x + 1,5x = 95,3.
№5. От мотка провода отрезали сначала 30%, а затем еще 60% остатка. После этого в мотке осталось 42 м провода. Сколько метров провода было в мотке первоначально?
№1. В железной руде содержится 45% железа. Сколько тонн железа содержится в 380 т руды?
№2. Найдите значение выражения
(299,3 : 14,6 – 9,62) 13 QUOTE 1415 3,5 + 72,2.
№3. За день вспахали 18% поля. Какова площадь всего поля, если вспахали 1170га?
№4.Решите уравнение 6,7y + 13 + 3,1y = 86,5.
№5. Израсходовали сначала 40% имевшихся денег, а затем еще 30% оставшихся. После чего осталось 105р. Сколько было денег первоначально?



К – 14. Измерение углов. Транспортир.

Вариант 1
Вариант 2

№1. Постройте углы, если:

№2. Начертите треугольник ANK такой, чтобы Измерьте и запишите градусные меры остальных углов треугольника.
№3. Луч OK делит прямой угол DOS на два угла так, что угол DOK составляет 0,7 угла DOS. Найдите градусную меру угла KOS.
№4. Развернутый угол AMF разделен лучом MC на два угла – АМС и CMF. Найдите градусные меры этих углов, если угол АМС вдвое больше угла CMF.
№5. Из вершины развернутого угла DKP проведены его биссектриса КВ и луч КМ так, что Какой может быть градусная мера угла DKM?

№1. Постройте углы, если:

№2. Начертите треугольник BCF такой, чтобы Измерьте и запишите градусные меры остальных углов треугольника.
№3. Луч AP делит прямой угол CAN на два угла так, что угол NAP составляет 0,3 угла CAN. Найдите градусную меру угла PAC.
№4. Развернутый угол BOE разделен лучом OT на два угла – BOT и TOE. Найдите градусные меры этих углов, если угол BOT втрое меньше угла TOE.
№5. Из вершины развернутого угла MNR проведены его биссектриса NВ и луч NP так, что Какой может быть градусная мера угла MNP?













К – 15. Итоговая контрольная работа
Вариант 1
Вариант 2

№1. Вычислите:

№2. Найдите значение выражения 13 QUOTE 1415 при x = 1, y = 0,1
№3. Решите уравнения:

№4. Расстояние между пунктами А и В равно 168 км. Товарный поезд проходит это расстояние за 2,8 ч, а скорый поезд – за 2,1 ч. Через сколько часов эти поезда встретятся, если одновременно выйдут навстречу друг другу из этих пунктов?
№5. Ребро одного куба в 3 раза больше ребра другого. Во сколько раз объем одного куба больше объема второго куба?

№1. Вычислите:

№2. Найдите значение выражения 13 QUOTE 1415 при x = 1, y = 0,1 №3. Решите уравнения:

№4. Маршрут длиной 105 км автобус проходит за 2,1 ч, а микроавтобус – за 1,5 ч. Через сколько часов после одновременного выхода в одном направлении микроавтобус обгонит автобус на 24 км?
№5. Ребро одного куба в 4 раза меньше ребра другого. Во сколько раз объем одного куба меньше объема второго куба?




























Тематические тесты.

Тест 1. Натуральные числа и шкалы.
Вариант 1.

ЧАСТЬ А
А1. Прочитайте число 2100391040 и укажите правильный ответ.
двадцать один миллион триста девяносто одна тысяча сорок
два миллиарда сто миллионов триста девяносто одна тысяча сорок
двадцать один миллион тридцать девять тысяч сто сорок
двадцать один миллиард три миллиона девяносто одна тысяча сорок
А2. Укажите большее из чисел:
1) 4200005 2) 429600 3) 420099 4) 4200010

А3. Выразите в миллиметрах 15 м 2 см 5 мм
1) 15025 2) 1525 3) 150025 4) 150205

А4. На каком рисунке правильно изображены точки М(4) и N(7) ?

А5. В числах 81***2 и 82***1 стерли несколько цифр и вместо них поставили звездочки. Если возможно, сравните эти числа и укажите правильный ответ.
1) 81***2 > 82***1 2) 81***2 = 82***1 3) 81***2 < 82***1 4) сравнить невозможно
А6. Какие точки удалены от точки В(5) на два единичных отрезка?

1) А и С 2) М и К 3) А и К 4) М и С
А7. На каком рисунке изображен луч?

А8. Каково значение числового выражения 13 EMBED Equation.DSMT4 1415
1) 22 2) 24 3) 4944 4) 4946

ЧАСТЬ B

В1. Прочитайте показания весов:




Вариант 2.


ЧАСТЬ А
А1. Прочитайте число 300301020 и укажите правильный ответ.
тридцать миллионов триста одна тысяча двадцать
три миллиарда триста одна тысяча двадцать
триста миллионов триста одна тысяча двадцать
три миллиарда три миллиона одна тысяча двадцать
А2. Укажите большее из чисел:
1) 5120005 2) 512600 3) 5120010 4) 512099

А3. Выразите в граммах 5 т 22 кг 4г
1) 5022004 2) 522004 3) 502204 4) 5022040

А4. На каком рисунке правильно изображены точки М(4) и N(8) ?
13 SHAPE \* MERGEFORMAT 1415
А5. В числах 73***1 и 72***8 стерли несколько цифр и вместо них поставили звездочки. Если возможно, сравните эти числа и укажите правильный ответ.
1) 73***1 > 72***8 2) 73***2 = 72***8 3) 73***1 < 72***8 4) сравнить невозможно
А6. Какие точки удалены от точки М(3) на два единичных отрезка?
1) К и С 2) В и К 3) А и В 4) только В
А7. На каком рисунке изображен отрезок?
А8. Каково значение числового выражения 13 EMBED Equation.DSMT4 1415
1) 256 2) 12312 3) 12420 4) 260

ЧАСТЬ B

В1. Прочитайте показания весов:





Тест 2


«Сложение и вычитание натуральных чисел»
Вариант 1.

ЧАСТЬ А

А1. Разложением какого числа на разрядные слагаемые является сумма 13 EMBED Equation.DSMT4 1415?
1) 30002040 2) 300002040 3) 300020040 4) 3000002040

А2. Вычислите: 13 EMBED Equation.DSMT4 1415
1) 976950 2) 281178 3) 2081178 4) 9976950

А3. Вычислите: 13 EMBED Equation.DSMT4 1415
1) 16572 2) 17472 3) 17562 4) 17572

А4. Найдите значение выражения: 13 EMBED Equation.DSMT4 1415 .
1) 31983 2) 32893 3) 31883 4) 32883

А5. Найдите значение выражения: 13 EMBED Equation.DSMT4 1415 .
1) 104 2) 100 3) 114 4) 94


А6. Не выполняя вычислений, выясните какое из выражений равно выражению 13 EMBED Equation.DSMT4 1415.

1) 13 EMBED Equation.DSMT4 1415 2) 13 EMBED Equation.DSMT4 1415
3) 13 EMBED Equation.DSMT4 1415 4) 13 EMBED Equation.DSMT4 1415

А7. Упростите выражение 13 EMBED Equation.DSMT4 1415
1) 13 EMBED Equation.DSMT4 1415 2) 13 EMBED Equation.DSMT4 1415 3) 13 EMBED Equation.DSMT4 1415 4) 13 EMBED Equation.DSMT4 1415


А8. Найдите значение выражения: 13 EMBED Equation.DSMT4 1415
1) 90 2) 100 3) 50 4) 110

ЧАСТЬ B

В1. В первом мешке было 53 кг зерна, что на 12 кг больше, чем во втором мешке. Сколько килограммов зерна было в обоих мешках вместе?







Вариант 2.

ЧАСТЬ А

А1. Разложением какого числа на разрядные слагаемые является сумма 13 EMBED Equation.DSMT4 1415?
1) 7000120 2) 70000120 3) 7000012 4) 70001020

А2. Вычислите: 13 EMBED Equation.DSMT4 1415
1) 416100 2) 416300 3) 417200 4) 416200

А3. Вычислите: 13 EMBED Equation.DSMT4 1415
1) 17075 2) 17705 3) 17065 4) 17085

А4. Найдите значение выражения: 13 EMBED Equation.DSMT4 1415 .
1) 260 2) 266 3) 256 4) 276

А5. Найдите значение выражения: 13 EMBED Equation.DSMT4 1415 .
1) 110 2) 120 3) 100 4) 50


А6. Не выполняя вычислений, выясните какое из выражений равно выражению 13 EMBED Equation.DSMT4 1415.

1) 13 EMBED Equation.DSMT4 1415 2) 13 EMBED Equation.DSMT4 1415
3) 13 EMBED Equation.DSMT4 1415 4) 13 EMBED Equation.DSMT4 1415

А7. Упростите выражение 13 EMBED Equation.DSMT4 1415
1) 13 EMBED Equation.DSMT4 1415 2) 13 EMBED Equation.DSMT4 1415 3) 13 EMBED Equation.DSMT4 1415 4) 13 EMBED Equation.DSMT4 1415


А8. Найдите значение выражения: 13 EMBED Equation.DSMT4 1415
1) 90 2) 100 3) 50 4) 60

ЧАСТЬ B

В1. В первом ящике было 24 кг конфет, что на 16 кг меньше, чем во втором ящике. Сколько килограммов конфет было в обоих ящиках вместе?










Тест 3
«Умножение и деление натуральных чисел»
Вариант1.

ЧАСТЬ А

А1. Вычислите 13 EMBED Equation.DSMT4 1415

1) 267950 2) 267860 3) 267960 4) 277960

А2. Вычислите: 13 EMBED Equation.DSMT4 1415

1) 75 2) 705 3) 805 4) 715

А3. Какой остаток может быть при делении числа на 86?

1) 88 2) 87 3) 86 4) 85

А4. Решите уравнение 13 EMBED Equation.DSMT4 1415 .

1) 104 2) 26 3) 8788 4) 52

А5. В выражении 13 EMBED Equation.DSMT4 1415 последним выполняется действие

1) деление 2) умножение 3) вычитание 4) сложение

А6. Упростите выражение 13 EMBED Equation.DSMT4 1415

1) 13 EMBED Equation.DSMT4 1415 2) 13 EMBED Equation.DSMT4 1415 3) 13 EMBED Equation.DSMT4 1415 4) 13 EMBED Equation.DSMT4 1415


А7. Вычислите 13 EMBED Equation.DSMT4 1415.

1) 18 2) 216 3) 36 4) 6


А8. Найдите значение выражения 13 EMBED Equation.DSMT4 1415.

1) 34 2) 14 3) 19 4) 16

ЧАСТЬ B

В1. Найдите уравнение для решения задачи: «В 13 EMBED Equation.DSMT4 1415 корзин насыпали по 12 кг груш, после чего осталось 5 кг груш. Сколько было корзин, если было 89 кг груш?»








Вариант 2.

ЧАСТЬ А

А1. Вычислите 13 EMBED Equation.DSMT4 1415

1) 108316 2) 107416 3) 108426 4) 108416

А2. Вычислите: 13 EMBED Equation.DSMT4 1415

1) 704 2) 714 3) 74 4) 694

А3. Какой остаток может быть при делении числа на 56?

1) 53 2) 56 3) 57 4) 61

А4. Решите уравнение 13 EMBED Equation.DSMT4 1415 .

1) 56 2) 49 3) 23 4) 59

А5. В выражении 13 EMBED Equation.DSMT4 1415 первым выполняется действие

1) вычитание 2) умножение 3) деление 4) сложение

А6. Упростите выражение 13 EMBED Equation.DSMT4 1415

1) 13 EMBED Equation.DSMT4 1415 2) 13 EMBED Equation.DSMT4 1415 3) 13 EMBED Equation.DSMT4 1415 4) 13 EMBED Equation.DSMT4 1415


А7. Вычислите 13 EMBED Equation.DSMT4 1415.

1) 131 2) 24 3) 21 4) 134


А8. Найдите значение выражения 13 EMBED Equation.DSMT4 1415.

1) 12 2) 47 3) 49 4) 51

ЧАСТЬ B

В1. Найдите уравнение для решения задачи: «В 13 EMBED Equation.DSMT4 1415 машин нагрузили по 4 т угля, после чего осталось 3 т угля. Сколько было машин, если было 51 т угля?»











Тест 4
«Площади и объемы»
Вариант1.

ЧАСТЬ А

А1. Найдите площадь квадрата, сторона которого равна 9 см.
1) 36 см2 2) 81 см 3) 81 см2 4) 18 см2

А2. Найдите площадь прямоугольника со сторонами 4 см и 8 см.
1) 32 см2 2) 24 см2 3) 80 см2 4) 32 см

А3. Найдите периметр прямоугольника, площадь которого равна 48 см2, а одна из его сторон - 4 см.
1) 64 см 2) 32см 3) 24 см 4) 52 см

А4. Найдите площадь фигуры, изображенной на рисунке.

1) 40 см2 2) 24 см2 3) 16 см2 4) 14 см2
А5. Найдите объем прямоугольного параллелепипеда, длина которого равна 8 см, ширина - 5 см, а высота - 4 см.

1) 120 см3 2) 60 см3 3) 160 см3 4) 22 см3

А6. Найдите объем куба с ребром, равным 6 см.
1) 40 см3 2) 16 см3 3) 216 см3 4) 12 см3

А7. Объем комнаты равен 72 м3. Длина комнаты 6 м, ширина 4 м. Найдите высоту комнаты.
1) 3 м 2) 2 м 3) 4м 4) 3 м2

А8. Найдите объем прямоугольного параллелепипеда, изображенного на рисунке.

1) 350 см3 2) 35 см3 3) 22 см3 4) 120 см3

ЧАСТЬ B

В1. Ширина прямоугольного параллелепипеда 9 см, и она меньше длины в 3 раза, но больше высоты на 6 см. Найдите сумму длин всех ребер








Вариант 2.

ЧАСТЬ А

А1. Найдите площадь квадрата, сторона которого равна 3 см.
1) 6 см2 2) 12 см 3) 12 см2 4) 9 см2

А2. Найдите площадь прямоугольника со сторонами 13 см и 5 см.
1) 36 см2 2) 65 см2 3) 18 см2 4) 65 см

А3. Найдите периметр прямоугольника, площадь которого равна 132 см2, а одна из его сторон - 12 см.
1) 23 см 2) 44 см 3) 46 см 4) 56 см

А4. Найдите площадь фигуры, изображенной на рисунке.
1) 21 см2 2) 24 см2 3) 26 см2 4) 19 см2
А5. Найдите объем прямоугольного параллелепипеда, длина которого равна 12 см, ширина - 5 см, а высота - 4 см.

1) 120 см3 2) 240 см3 3) 160 см3 4) 220 см3

А6. Найдите объем куба с ребром, равным 7 см.
1) 98 см3 2) 49 см3 3) 21 см3 4) 343 см3

А7. Объем комнаты равен 105 м3. Длина комнаты 7 м, ширина 5 м. Найдите высоту комнаты.
1) 2м 2) 3 м 3) 4м 4) 3 м2

А8. Найдите объем прямоугольного параллелепипеда, изображенного на рисунке.

1) 180 см3 2) 90 см3 3) 900 см3 4) 16 см3

ЧАСТЬ B

В1. Длина прямоугольного параллелепипеда 8 см, и она больше ширины в 2 раза, но меньше высоты на 5 см. Найдите сумму длин всех ребер








Тест 5
«Обыкновенные дроби»
Вариант 1.

ЧАСТЬ А
А1. На каком рисунке закрашена 13 EMBED Equation.DSMT4 1415 часть фигуры?






А2. Выразите в минутах 10 сек.
1) 13 EMBED Equation.DSMT4 1415 2) 13 EMBED Equation.DSMT4 1415 3) 13 EMBED Equation.DSMT4 1415 4) 13 EMBED Equation.DSMT4 1415
А3. Какая точка имеет координату 13 EMBED Equation.DSMT4 1415?

1) A 2) B 3) C 4) D
А4. Сравните числа : 13 EMBED Equation.DSMT4 1415 .
1) 13 EMBED Equation.DSMT4 1415 2) 13 EMBED Equation.DSMT4 1415 3) 13 EMBED Equation.DSMT4 1415 4) 13 EMBED Equation.DSMT4 1415
А5. Представьте в виде неправильной дроби 13 EMBED Equation.DSMT4 1415.
1) 13 EMBED Equation.DSMT4 1415 2) 13 EMBED Equation.DSMT4 1415 3) 13 EMBED Equation.DSMT4 1415 4) 13 EMBED Equation.DSMT4 1415
А6. Вычислите 13 EMBED Equation.DSMT4 1415 . 1) 13 EMBED Equation.DSMT4 1415 2) 13 EMBED Equation.DSMT4 1415 3) 13 EMBED Equation.DSMT4 1415 4) 13 EMBED Equation.DSMT4 1415
А7. Длина прямоугольника 13 EMBED Equation.DSMT4 1415 м, а ширина на 13 EMBED Equation.DSMT4 1415м меньше. Какова ширина прямоугольника?
1) 13 EMBED Equation.DSMT4 1415 2) 13 EMBED Equation.DSMT4 1415 3) 13 EMBED Equation.DSMT4 1415 4) 13 EMBED Equation.DSMT4 1415
А8. Вычислите 13 EMBED Equation.DSMT4 1415 . 1) 13 EMBED Equation.DSMT4 1415 2) 13 EMBED Equation.DSMT4 1415 3) 13 EMBED Equation.DSMT4 1415 4) 13 EMBED Equation.DSMT4 1415

ЧАСТЬ B В1. Решите уравнение 13 EMBED Equation.DSMT4 1415
Вариант 2.




А1. Какая часть фигуры, изображенной на рисунке закрашена?
1) 13 EMBED Equation.DSMT4 1415 2) 13 EMBED Equation.DSMT4 1415 3) 13 EMBED Equation.DSMT4 1415 4) 13 EMBED Equation.DSMT4 1415

А2. Выразите в часах 5 мин.
1) 13 EMBED Equation.DSMT4 1415 2) 13 EMBED Equation.DSMT4 1415 3) 13 EMBED Equation.DSMT4 1415 4) 13 EMBED Equation.DSMT4 1415

А3. Какая точка имеет координату 13 EMBED Equation.DSMT4 1415?

1) A 2) B 3) C 4) D
А4. Сравните числа : 13 EMBED Equation.DSMT4 1415 .
1) 13 EMBED Equation.DSMT4 1415 2) 13 EMBED Equation.DSMT4 1415 3) 13 EMBED Equation.DSMT4 1415 4) 13 EMBED Equation.DSMT4 1415
А5. Представьте в виде смешанного числа 13 EMBED Equation.DSMT4 1415.
1) 13 EMBED Equation.DSMT4 1415 2) 13 EMBED Equation.DSMT4 1415 3) 13 EMBED Equation.DSMT4 1415 4) 13 EMBED Equation.DSMT4 1415

А6. Вычислите 13 EMBED Equation.DSMT4 1415 . 1) 13 EMBED Equation.DSMT4 1415 2) 13 EMBED Equation.DSMT4 1415 3) 13 EMBED Equation.DSMT4 1415 4) 13 EMBED Equation.DSMT4 1415
А7. В ведро входит 13 EMBED Equation.DSMT4 1415 литра воды, а в бидон на 13 EMBED Equation.DSMT4 1415литра меньше. Сколько литров воды входит в бидон?
1) 13 EMBED Equation.DSMT4 1415 2) 13 EMBED Equation.DSMT4 1415 3) 13 EMBED Equation.DSMT4 1415 4) 13 EMBED Equation.DSMT4 1415
А8. Вычислите 13 EMBED Equation.DSMT4 1415 . 1) 13 EMBED Equation.DSMT4 1415 2) 13 EMBED Equation.DSMT4 1415 3) 13 EMBED Equation.DSMT4 1415 4) 13 EMBED Equation.DSMT4 1415


ЧАСТЬ B

В1. Решите уравнение 13 EMBED Equation.DSMT4 1415


Тест 6
«Десятичные дроби»

Вариант 1.

ЧАСТЬ А

А1. Запишите в виде десятичной дроби число 13 EMBED Equation.DSMT4 1415.
1) 4,0043 2) 4,043 3) 4,43 4) 4,00043

А2. Назовите большее из чисел:

1) 9,070 2) 9, 7 3) 9,698 4) 9,007

А3. Вычислите: 13 EMBED Equation.DSMT4 1415.

1) 97,88 2) 20,93 3) 20,093 4) 20,903

А4. Вычислите: 13 EMBED Equation.DSMT4 1415.
1) 6,035 2) 6,235 3) 6,335 4) 6,245

А5. Укажите до какого разряда округлены числа 13 EMBED Equation.DSMT4 1415.

1) а) до десятых б) до сотых 2) а) до сотых б) до сотых 3) а) до сотых б) до десятых 4) а) до сотых б) до тысячных

А6. Турист прошел за 6 часов 32,1 км. Сколько километров пройдет турист с такой же скоростью за 7 часов?

1) 37,35 2) 37,5 3) 37,45 4) 37,75

А7. Представьте в виде десятичной дроби 13 EMBED Equation.DSMT4 1415
1) 0,0375 2) 0,35 3) 0,75 4) 0,375


А8. Найдите значение выражения: 13 EMBED Equation.DSMT4 1415.

1) 1,337 2) 1,0625 3) 1,895 4) 12,812


ЧАСТЬ B

В1. Площадь поля 8,7 га. Тракторист вспахал 0,7 площади поля. Сколько гектаров ему осталось вспахать?






Вариант 2.


ЧАСТЬ А

А1. Запишите в виде десятичной дроби число 13 EMBED Equation.DSMT4 1415
1) 1, 0027 2) 1,000027 3) 1,00027 4) 1,27

А2. Назовите большее из чисел:

1) 10, 8 2) 10,080 3) 10,798 4) 10,008

А3. Вычислите: 13 EMBED Equation.DSMT4 1415

1) 36,773 2) 36,783 3) 84,6 4) 36,6

А4. Вычислите: 13 EMBED Equation.DSMT4 1415.

1) 2,05 2) 2,5 3) 2,005 4) 1,05

А5. Укажите до какого разряда округлены числа 13 EMBED Equation.DSMT4 1415.

1) а) до десятых б) до сотых 2) а) до единиц б) до десятых 3) а) до единиц б) до десятых 4) а) до единиц б) до сотых

А6. Мотоциклист проехал за 7 часов 387,8 км. Сколько километров проедет мотоциклист с такой же скоростью за 9 часов?

1) 398,6 2) 488,6 3) 498,6 4) 498,06
А7. Представьте в виде десятичной дроби 13 EMBED Equation.DSMT4 1415
1) 0,325 2) 0,3125 3+) 0,375 4) 0,315


А8. Найдите значение выражения: 13 EMBED Equation.DSMT4 1415
1) 1,224 2) 12,24 3) 12,4 4) 12,12

ЧАСТЬ B

В1. В коробке было 4,3 кг конфет. Продали 0,7 содержимого коробки. Сколько килограммов конфет осталось в коробке?










Тест 7
«Итоговое повторение»

Вариант 1

ЧАСТЬ А
А1. Представьте в виде неправильной дроби 13 EMBED Equation.DSMT4 1415.
1) 13 EMBED Equation.DSMT4 1415 2) 13 EMBED Equation.DSMT4 1415 3) 13 EMBED Equation.DSMT4 1415 4) 13 EMBED Equation.DSMT4 1415
А2. Вычислите 13 EMBED Equation.DSMT4 1415
1) 38 2) 308 3) 3008 4) 380
А3. Вычислите: 13 EMBED Equation.DSMT4 1415.
1) 602,75 2) 603,75 3) 52,68 4) 526,8
А4. На каком рисунке правильно изображены точки М(3) и N(8) ?
13 SHAPE \* MERGEFORMAT 1415
А5. Вычислите: 13 EMBED Equation.DSMT4 1415. 1) 0,904 2) 9,4 3) 0,94 4) 0,094
А6. Представьте в виде десятичной дроби 13 EMBED Equation.DSMT4 1415.
1) 0,075 2) 0,025 3) 0,75 4) 0,375
А7. Округлите 13 EMBED Equation.DSMT4 1415 до десятых.
1) 2,4 2) 2,34 3) 2,35 4) 2,3
А8. Расположите в порядке убывания числа 13 EMBED Equation.DSMT4 1415
1) 13 EMBED Equation.DSMT4 1415 2) 13 EMBED Equation.DSMT4 1415
3) 13 EMBED Equation.DSMT4 1415 4) 13 EMBED Equation.DSMT4 1415
А9. Продолжительность фильма 13 EMBED Equation.DSMT4 1415ч., а спектакля на 13 EMBED Equation.DSMT4 1415ч. – больше. Сколько времени длился спектакль?
1) 13 EMBED Equation.DSMT4 1415 2) 13 EMBED Equation.DSMT4 1415 3) 13 EMBED Equation.DSMT4 1415 4) 13 EMBED Equation.DSMT4 1415
А10. Решите уравнение 13 EMBED Equation.DSMT4 1415
1) 8,5 2) 3,4 3) 2,36 4) 6

ЧАСТЬ B

В1. На изготовление детали требуется 0,16 кг стали. Сколько деталей изготовят из 11,2 кг стали?
В2. Решите уравнение 13 EMBED Equation.DSMT4 1415.


Вариант 2.

ЧАСТЬ А
А1. Представьте в виде смешанного числа 13 EMBED Equation.DSMT4 1415.
1) 13 EMBED Equation.DSMT4 1415 2) 13 EMBED Equation.DSMT4 1415 3) 13 EMBED Equation.DSMT4 1415 4) 13 EMBED Equation.DSMT4 1415
А2. Вычислите 13 EMBED Equation.DSMT4 1415
1) 784740 2) 774740 3) 784760 4) 784730
А3. Вычислите: 13 EMBED Equation.DSMT4 1415.
1) 32,73 2) 33,73 3) 32,83 4) 31,73
А4. На каком рисунке правильно изображены точки М(4) и N(8) ?
13 SHAPE \* MERGEFORMAT 1415
А5. Вычислите: 13 EMBED Equation.DSMT4 1415.
1) 0,207 2) 2,07 3) 2,7 4) 0,27
А6. Представьте в виде десятичной дроби 13 EMBED Equation.DSMT4 1415.
1) 0,075 2) 0,0825 3) 0,0875 4) 0,875
А7. Округлите 13 EMBED Equation.DSMT4 1415 до тысячных.
1) 5,247 2) 5,248 3) 5,2475 4) 5,25
А8. Укажите большее число:
1) 13 EMBED Equation.DSMT4 1415 2) 13 EMBED Equation.DSMT4 1415 3) 13 EMBED Equation.DSMT4 1415 4) 13 EMBED Equation.DSMT4 1415
А9. В одном пакете 13 EMBED Equation.DSMT4 1415кг печенья, а в другом на 13 EMBED Equation.DSMT4 1415кг меньше. Сколько килограммов печенья во втором пакете?
1) 13 EMBED Equation.DSMT4 1415 2) 13 EMBED Equation.DSMT4 1415 3) 13 EMBED Equation.DSMT4 1415 4) 13 EMBED Equation.DSMT4 1415
А10. Решите уравнение 13 EMBED Equation.DSMT4 1415
1) 0,3 2) 0,22 3) 0,2 4) 0,02

ЧАСТЬ B

В1. В один подарочный пакет укладывается 0,6 кг конфет. Сколько пакетов необходимо для 21,6 кг конфет?
В2. Решите уравнение 13 EMBED Equation.DSMT4 1415.





При составлении контрольных работ использовался сборник: Ершова А.П., Голобородько В.В. «Самостоятельные и контрольные работы»,5 класс. – М.: 2010.

САМОСТОЯТЕЛЬНЫЕ РАБОТЫ

Проценты и углы.
Вариант 1
Вариант 2

№1. Рабочий по плану должен изготовить 250 деталей, но он перевыполнил план на 12%. Сколько деталей изготовил рабочий?
№2. На клумбе посадили 36 луковиц тюльпанов, что составило 48% всех купленных луковиц. Сколько всего купили луковиц тюльпанов?
№3.Луч NP делит развернутый угол MNK на два угла так, что градусная мера угла MNP в три раза меньше градусной меры угла PNK. Чему равны градусные меры углов MNP и PNK?
№4. Постройте треугольник ABC так, что угол A равен 45°, а угол B равен 99°. Какова градусная мера угла C? Чему равна сумма углов треугольника? Какой процент от суммы углов треугольника составляет угол C?
№5. Число А на 20% больше числа В. Во сколько раз число А больше числа В?
№1. Тракторист должен вспахать 120 га земли, но он перевыполнил план на 15%. Сколько гектаров земли вспахал тракторист?
№2. Утром рыбак поймал 35 карасей, что составило 28% улова карасей за день. Сколько всего карасей поймал рыбак за день?
№3. Луч NP делит развернутый угол MNK на два угла так, что градусная мера угла MNP в пять раз больше градусной меры угла PNK. Чему равны градусные меры углов MNP и PNK?
№4. Постройте треугольник ABC так, что угол A равен 35°, а угол B равен 100°. Какова градусная мера угла C? Чему равна сумма углов треугольника? Какой процент от суммы углов треугольника составляет угол C?
№5. Число А на 20% меньше числа В. Во сколько раз число А меньше числа В?
















Дробные числа.
Вариант 1
Вариант 2

№1. Выполните действия:

№2. Упростите выражение 2,3x – 0,03x + 0,82x. Вычислите при x = 32; 10; 0,1.
№3.Автомобиль проехал 92,4 км. Ему осталось проехать 13 QUOTE 1415 всего пути. Сколько километров осталось проехать автомобилю?
№4. Собственная скорость катера равна 32 км/ч, а скорость течения – 2,1 км/ч. Сначала катер шел 0,7 ч по течению, а затем 1,7 ч против течения. Сколько всего километров прошел катер по реке?
№5. Даны два числа. Первое число составляет 12% от 1200 и 24% от второго числа. Найдите среднее арифметическое этих двух чисел.
№1. Выполните действия:

№2. Упростите выражение 4,7x – 0,07x + 0,48x. Вычислите при x = 32; 10; 0,1.
№3. Грузовик проехал 13 QUOTE 1415 всего пути. Ему осталось проехать 91,2 км. Сколько километров проехал грузовик?
№4. Собственная скорость парохода равна 36 км/ч, а скорость течения – 2,3 км/ч. Сначала пароход шел 1,6 ч против течения, а затем 0,6 ч по течению. Сколько всего километров прошел пароход по реке?
№5. Даны два числа. Второе число составляет 80% от 8000 и 40% от первого числа. Найдите среднее арифметическое этих двух чисел.
































13PAGE 15


13PAGE 14115



1)

M

N

1

0

3)

M

N

1

0

2)

M

N

1

0

4)

M

N

1

0

A

C

B

M

K

1

0

A

B

1)

2)

3)

4)

A

B

A

B

A

B

3 см

2 см

5 см

4 см

7 см

5 см

1 дм

3 см

5 см

6 см

2 см

9 см

5 см

2 дм

D

С

В

А

1

0

D

С

В

А

1

0

1)

M

N

1

0

3)

M

N

1

0

2)

M

N

1

0

4)

M

N

1

0

1)

M

N

1

0

3)

M

N

1

0

2)

M

N

1

0

4)

M

N

1

0